General Chemistry 1 2nd Quarter

You might also like

Download as pdf or txt
Download as pdf or txt
You are on page 1of 84

WEEKLY LEARNING ACTIVITY SHEETS

General Chemistry 1, Quarter 2, Week 1

THE QUANTUM MECHANICAL DESCRIPTION OF THE ATOM


AND ITS ELECTRONIC STRUCTURE

Name: ___________________________________________ Section: ____________________

Learning Objectives:

 Most Essential Learning Competencies:


- Use quantum numbers to describe an electron in an atom
(STEM_GC11ESIIa-b-54)
- Determine the magnetic property of the atom based on its electronic configuration
(STEM_GC11ESIIa-b-57)
- Draw an orbital diagram to represent the electron configuration of atoms
(STEM_GC11ESIIa-b-58)
 Specific Objectives:
1. Describe electrons using quantum numbers
2. Draw orbital diagrams to represent the electron configuration of atoms
3. Determine whether an atom is paramagnetic or diamagnetic based on its electron
configuration
4. Create models depicting the shape of the s, p, and d orbitals
5. Cite practical applications of quantum numbers/orbital diagrams/magnetic
property in real life
Time Allotment: 4 hours

Key Concepts
A. Quantum Numbers:
 Determine the home address of your friends or relatives using the format of house number,
street, purok name, barangay, municipality and zip code. How many of your
friends/relatives live on the same street? How many have the same house number?
Just as there are no two houses that have the same address (they could have the same
purok but they are of different house number/street), no two electrons in an atom have the
same set of four quantum numbers. Quantum numbers tell where an electron is located
around the nucleus of an atom. It is kind of like the address of electrons.
 There are four quantum numbers, three of which, the principal quantum number (n), the
angular momentum quantum number (ℓ), and the magnetic quantum number (ml)
describe the atomic orbitals that is a region of space where you can most probably find the
electron. A fourth quantum number, the spin quantum number (ms) completes the
description (spin orientation) of the electrons in the atoms.
 The Principal Quantum Number (n)
a. Determines the energy of an orbital; describes the
energy level an electron is placed in
b. Determines the orbital size
c. Is associated with the average distance of the electron
from the nucleus in a particular orbital; the larger the
value of n, the farther the average distance of the
electron from the nucleus
d. n can have the values 1, 2, 3, …

Author: Ryan Paul M. Vales


School/Station: Tagbina National High School
Division: Surigao del Sur
email address: ryanpaul.vales@deped.gov.ph
e. Orbitals with the same principal quantum number (n) are said to be in the same
shell.
 The Angular Momentum Quantum Number (ℓ)
a. Describes the “shape” of the orbitals
b. ℓ can have the values 0, 1, 2, up to n-1.
So if the principal quantum number (n) is 2, ℓ could be 0 and 1. Why 1? study the
calculation below:
ℓ=n-1
ℓ=2-1
ℓ=1
c. Orbitals that have the same n and ℓ values belong to the same subshell.

d. It is usually designated by letters s, p, d, f, … which have a historical origin from


spectral lines. The s, p, d, f designations of the orbitals refer to sharp, principal,
diffuse, and fundamental lines in emission spectra.
 The Magnetic Quantum Number (ml)
a. Describes the orientation of the orbital in space
b. Describes the number of orbitals within a subshell
c. ml can have the values -ℓ up to +ℓ:
- ℓ, (-ℓ + 1), … 0, … (+ ℓ -1), + ℓ
The following table summarizes the relationship of n, ℓ, and ml:

Author: Ryan Paul M. Vales


School/Station: Tagbina National High School
Division: Surigao del Sur
email address: ryanpaul.vales@deped.gov.ph
 The Electron Spin Quantum Number (ms)
a. The first three quantum numbers (n, ℓ, ml) describe the energy, shape and
orientation of the orbitals. The 4th quantum number (ms) refers to two different spin
orientations of electrons in an orbital.
b. In 1925, Uhlenbeck, Goudsmit, and Kronig introduced the idea of the self-rotation
of the electron. The spin orientations are assigned the number ms = ½, ms = -½
which are called “spin-up” and “spin-down”, respectively.
c. The electrons are paired such that one spins upward and one downward. This
neutralizes the effect of their spin on the action of the atom as a whole. But in the
valence shell of atoms where there is a single electron whose spin remains
unbalanced, the unbalanced spin creates spin magnetic moment, making the
electron act like a very small magnet.
 The Representations of the Shapes of Atomic Orbitals
a. Strictly speaking, an orbital does not have a definite shape, but the electron’s probability
of being in a certain place has a shape.
While electron can be found anywhere
around the nucleus, there are regions
where there is much higher probability
of finding it. For s orbital, the probability
distribution is spherical as shown in
figure a. All the s orbitals are spherical
in shape but they differ in size, which
increases as the value of the principal quantum number (n) increases.
b. There are three 2p orbitals: 2px, 2py, 2pz indicating the axes along which they are
oriented. The electron probability density of p orbitals is not spherically symmetric; it
has a double teardrop shape or a dumbbell shape (as described in some books). The
electron can most probably be found
within the two lobes of the dumbbell
region; there is a zero probability of
finding it along the nodal planes found
in the axes. All three 2p orbitals are
identical in shape and energy but differ
in orientation as shown in figure (b).
The p orbitals of higher principal
quantum numbers have similar shapes.
c. Figure c shows that d orbitals occur for
the first time when n = 3; d orbitals have
five orientations. Four of the d orbitals
have identical basic shapes except for
their orientation with respect to the
axes. The wave functions exhibit
positive and negative lobes along the
axes; there is a zero probability of
finding the electron at the origin. The
fifth wave function, dz2, has a similar
shape with that of the p-orbital with a
donut-shape region along the x-axis.

Image Source:
(a-c) Wyona C Patalinghug et al., Teaching Guide for Senior High School General Chemistry 1 (Diliman, Quezon City,
Philippines: Commission on Higher Education, 2016), 171-172.

 The four quantum numbers compose the numbers that describe where an electron is
located around the nucleus of an atom. It is kind of like the electron's address. The
quantum numbers shall be in the order: energy level (n), sub-level or orbital type (ℓ), the

Author: Ryan Paul M. Vales


School/Station: Tagbina National High School
Division: Surigao del Sur
email address: ryanpaul.vales@deped.gov.ph
orientation of the orbital specified in (mℓ), and the orientation of the spin of the electron
(ms). It is written in the order (n, ℓ, mℓ, ms).
 Example 1: What is the allowed set of quantum numbers for an electron that is
found in the first energy level?
a. The energy level, n = 1.
b. The orbital type is only s, thus, ℓ = 0
c. From ℓ, the orbital type is s. An s orbital has only one orientation, designated as
0, so, mℓ = 0
d. In the 1s orbital, an electron can have an up-spin or a down-spin. Therefore, ms
could be 1/2 or -1/2.
The allowed set of quantum numbers for 1s electron are (1,0,0,1/2) and (1,0,0,-
1/2).
How does (1,0,0,1/2) differ from (1,0,0,-1/2)? The first set corresponds to the
electron with spin up and the second set refers to the electron with spin down.
 Example 2: Given the following information for the three electrons:
Quantum Number Electron A Electron B Electron C
n 4 5 4
ℓ 2 0 2
mℓ -2 0 +1
ms 1/2 1/2 -1/2
Describe the electrons based on quantum numbers.
Answer: Electron A first electron in the 4d orbital (4d1)
Electron B first electron in the 5s orbital (5s1)
Electron C ninth electron in the 4d orbital (4d9)
 Electron Configuration
The distribution of electrons among the orbitals of an atom is known as electron
configuration. The order in which the electrons are filled in can be read from the periodic table of
elements in the following manner:

Image source: Kevin Boudreaux, “Quantum Numbers, Atomic Orbitals, and Electron Configurations,”
2020, https://www.angelo.edu/faculty/kboudrea/index.htm.

Author: Ryan Paul M. Vales


School/Station: Tagbina National High School
Division: Surigao del Sur
email address: ryanpaul.vales@deped.gov.ph
- You can refer to the periodic table presented above to assign electron configurations for
each element. To write the electron configurations, indicate the orbitals that are occupied
by electrons, then write a superscript that indicates how many electrons are in the set of
orbitals. For instance, write the electron configuration of lithium:
Lithium has an atomic number of 3. The atom’s atomic number is the number of protons
of the atom; thus, it is also the number of electrons in an atom with 0 charge. So for lithium,
it has 3 electrons in its ground state. The 1st and 2nd electrons are in 1s orbital and the 3rd
electron is in 2s orbital, so its electron configuration is 1s 22s1.
- Condensed electron configuration can be used to shorten the process of writing a long
electron configuration. To do this, write the symbol of the nearest noble gas with fewer
electrons than your atom in brackets, after which, continue with the electron configuration
for the orbital sets. For instance, the electron configuration of lithium which is 1s 22s1 can
also be written as [He]2s1. Helium is the nearest noble gas with fewer electrons than lithium.
- Caution: the d and f orbital regions in the periodic table correspond to energy levels that
are different from the period they're located in. Notice that the first row of the d orbital block
corresponds to the 3d orbital even if it is in period 4, while the first row of the f orbital
corresponds to the 4f orbital even if it is in period 6.

B. Orbital Diagrams:

 Writing the quantum numbers of electrons in set notation like (3,1,-1,-1⁄2) is time
consuming and difficult to compare so an abbreviated form was developed. With electron
configuration, the first two quantum numbers, n and ℓ, are listed; it also shows how many
electrons exist in each orbital. Many times it is needed to see all the quantum numbers in
an electron configuration, this is the purpose of the orbital diagram. Orbital diagrams
pictorially describe the electrons in an atom. In addition to listing the principal quantum
number, n, and the subshell, ℓ, (which can also be seen in the electron configuration) the
orbital diagram shows all the different orientations and the spin of every electron. Orbital
diagrams illustrate the number of subshells using lines or boxes for electrons (one box/line
for s-orbital, three for p-orbitals, five for d-orbitals, and 7 for f-orbitals). In each box,
electron spin is noted by using arrows; up arrows indicate 1⁄2 spin and down arrows mean
–1⁄2 spin.

 Rules for Filling Orbitals:


a. Aufbau Principle states that the lowest energy orbital is filled first. Electrons usually
fill the lowest energy level and the simplest orbital shape first before the higher energy
orbitals are filled. The order of filling based on the Aufbau Principle is presented in
the following diagram.

Author: Ryan Paul M. Vales


School/Station: Tagbina National High School
Division: Surigao del Sur
email address: ryanpaul.vales@deped.gov.ph
Figure f. The order of orbital filling based on the Aufbau Principle

- Based on figure f, examine why the first orbital diagram below violates Aufbau
Principle.

- The Aufbau Principle is not a universal rule; not all atoms obey it. Around ten
transition metals violate the Aufbau Principle (Cr, Cu, Nb, Mo, Ru, Rh, Pd, Ag, Pt,
and Au). In each element, the d orbital had an extra electron from the s orbital,
except in Pd where two electrons are consumed by the d orbital. In lanthanides
and actinides, ten elements go against the Aufbau Principle (La, Ce, Gd, Ac, Th,
Pa, U, Np, Cm, and Lr). In the said elements, the d orbital takes an electron from
the f orbital; Th and Lr are special cases. In Th, 6d takes both electrons from 5f
while in Lr 6d is replaced by 7p.

b. Pauli Exclusion Principle states that no two electrons in an atom can have the
same four quantum numbers. This is why each orbital only has two electrons, one
with up-spin (1⁄2) and one with down-spin (–1⁄2).

c. Hund’s Rule states that same-energy orbitals, those which differ only in their
orientation, are filled with electrons that have the same spin before the second
electron is added to any of the orbitals.

Author: Ryan Paul M. Vales


School/Station: Tagbina National High School
Division: Surigao del Sur
email address: ryanpaul.vales@deped.gov.ph
Example:
What element has the following quantum numbers for their last electron (in the
subshell with highest energy level):
(a) 3,1,1,1/2; (2) 1,0,0,-1/2?
Illustrate the orbital diagram of the element.
a) The quantum numbers 3,1,1,1/2 corresponds to Phosphorus. n indicates that
the element has 3 energy levels, the element is in the 3rd period in the periodic
table; ℓ indicates that the orbital type is p, the mℓ is equal to 1 suggesting that all
3 orientations of the p-orbital are occupied; ms is equal to 1/2 indicating up-spin
of the last electron and that the last orbital only has 1 electron, all orientations
of the p-orbital have unpaired electrons. With this, it can be deduced that the
valence subshell of the element is 3p3, the only element in the 3 rd period of the
periodic table having this valence subshell is phosphorus.

b) The quantum numbers 1,0,0,-1/2 corresponds to Helium. n indicates that the


element has 1 energy level, the element is in the 1st period in the periodic table;
ℓ indicates that the orbital type is s, there is only one orientation of the s orbital
thus mℓ=0; ms is equal to -1/2 indicating down-spin of the last electron and that
the last orbital has 2 electrons. With this, it can be deduced that the
valence subshell of the element is 1s2, the only element in the 1st period
of the periodic table having this valence subshell is helium.

C. Paramagnetism and Diamagnetism


 Paramagnetism refers to the magnetic state of an atom that has one or more unpaired
electrons. Due to the electrons' magnetic dipole moments, the unpaired electrons are
attracted by a magnetic field. Hund's Rule states that electrons have to occupy every orbital
singly before any orbital is doubly occupied. This could leave the atom with many unpaired
electrons. Since unpaired electrons can spin in either direction, they exhibit magnetic
moments in any direction. This capability lets paramagnetic atoms to be attracted to
magnetic fields.
 Diamagnetic substances are characterized by paired electrons. According to
the Pauli Exclusion Principle, no two electrons could occupy identical quantum state at the
same time, the electron spins are oriented in opposite directions. With this, the magnetic
fields of the electrons cancel out; thus there is no net magnetic moment, and the atom
cannot be attracted into a magnetic field.
 How to tell if a substance is paramagnetic or diamagnetic:
Electron configuration can be used to determine the magnetic properties of an element: If
it has unpaired electrons, then the substance is paramagnetic and if all electrons are paired,
the substance is diamagnetic. This process can be broken into these steps:
1. Write down the electron configuration
2. Draw the valence orbitals
3. Identify if unpaired electrons exist
4. Based on the orbital diagram, determine whether the element is diamagnetic or
paramagnetic

Author: Ryan Paul M. Vales


School/Station: Tagbina National High School
Division: Surigao del Sur
email address: ryanpaul.vales@deped.gov.ph
 Example: Zinc Atoms
o Step 1: Determine the electron configuration
For Zn atoms, the electron configuration is [Ar]4s 23d10
o Step 2: Draw the valence orbitals
Focus on the valence electrons only (ignore the core electrons).

o Step 3: Look for unpaired electrons


There are no unpaired electrons.
o Step 4: Determine whether the substance is diamagnetic or paramagnetic
Since there are no unpaired electrons, zinc atoms are diamagnetic.

Activity No. 1: Complete me!


What you need: Pen, Paper, Periodic Table of Elements
What to do: Look at the given elements on the periodic table. Write the electron configuration in
the second column. In the third column, draw the orbital diagram of the subshell in the highest
energy level containing the last electron. In the fourth column, determine whether it is
paramagnetic or diamagnetic. Lastly, assign quantum numbers as directed. Write your answers
on a separate sheet of paper.
Draw the orbital Quantum Number of the
Condensed diagram of the Is it Last Electron
Electron subshell in the paramagnetic
Element
Configu- highest energy or
N ℓ ml ms
ration level containing diamagnetic?
the last electron
Na [Ne]3s1 ↑ paramagnetic 3 0 0 1/2
Ca
Cl
Sc
Zn
O

Activity No. 2: Make me!


- What you need: paper, pen, scissors/cutter, scotch tape, sticks, cardboard or acetate,
styrofoam balls or balloons or any locally available materials that could depict an orbital shape
such as mansanitas, and the likes.
- What to do: In this activity, you will be crafting models depicting the shape of the s, p,
and d orbitals. Follow the steps below:
1. Create a model of the s, p, and d orbitals using locally available materials. You may
refer to the key concepts section for the shapes of the orbitals. Use the
styrofoam/balloon/other materials to depict the region where you can most likely
find an electron. Use the cardboard/acetate and sticks to depict the nodal planes.
2. Label your model using strips of paper.
3. If you are using perishable items such as fruits for your models, take pictures of your
model. There should be pictures where you are holding your output and other

Author: Ryan Paul M. Vales


School/Station: Tagbina National High School
Division: Surigao del Sur
email address: ryanpaul.vales@deped.gov.ph
pictures showing close-up look of your model. Print the pictures and attach them in
your activity sheet. If you don’t have printers at home, you may send the pictures in
your class’ group chat instead.
4. If you are using non-perishable items for your models, you may submit the model
itself to your teacher during the retrieval of modules/activity sheets. Make sure to
pack the model securely to keep it intact.
- Guide Questions:
1. Describe the shape of the s, p, and d orbitals.
2. Can electrons be exactly found on the regions depicted by the orbital shapes?
Explain.

Rubric for Model of Orbital Shapes


4 3 2 1
Model Model is Model is well Model shows Model is not
completely thought out and evidence of finished and is
finished and is implemented research in not well thought
well thought out. It correctly but it has a planning but it out. It has lots
accurately depicts missing component. has missing of errors.
the orbital shapes. It contains one or components. It
two errors. contains some
errors.
Construction Model is carefully Model is carefully The model is The model is
constructed using constructed using constructed but poorly
appropriate locally appropriate locally uses some constructed and
available materials available materials inappropriate is messy.
that showcase but it does not materials. It is
student’s showcase student’s neat.
creativity. It is creativity. It is neat.
neat.
Label All are labeled 1-2 parts of the label There are labels No label, or
correctly. are not formatted but most parts there are labels
correctly. are formatted but are
incorrectly. formatted
incorrectly.

Reflection
Give practical applications of quantum numbers/orbital diagrams/magnetic property in
real life. Write your responses on a separate sheet of paper. Be guided with the following rubrics:
_______________________________________________________________________________________________
_______________________________________________________________________________________________
_______________________________________________________________________________________________
_______________________________________________________________________________________________
_______________________________________________________________________________________________
_______________________________________________________________________________________________
_______________________________________________________________________________________________
_______________________________________________________________________________________________
_______________________________________________________________________________________________
_______________________________________________________________________________________________
_______________________________________________________________________________________________
_______________________________________________________________________________________________
_______________________________________________________________________________________________
_______________________________________________________________________________________________
_______________________________________________________________________________________________

Author: Ryan Paul M. Vales


School/Station: Tagbina National High School
Division: Surigao del Sur
email address: ryanpaul.vales@deped.gov.ph
Reflection Rubric
3 Practical application is scientifically explained consistent to the
concepts, and has no misconception.
2 Practical application is scientifically explained consistent to the
concepts, but with minimal misconception.
1 Practical application is explained consistent to the concepts, but
with misconceptions.
0 No discussion

References for learners:


Boudreaux, Kevin. “Quantum Numbers, Atomic Orbitals, and Electron Configurations,” 2020.
https://www.angelo.edu/faculty/kboudrea/index.htm.
Chang, Raymond. Chemistry. 10th ed. Boston: McGraw-Hill, 2010.
ChemistryGod. “Aufbau Principle with Exceptions ~ ChemistryGod,” February 18, 2020.
https://chemistrygod.com/aufbau-principle.
Clark, Jim, and Richard Spinney. “Magnetic Properties.” Chemistry LibreTexts. Libretexts, July
14, 2020.
https://chem.libretexts.org/Bookshelves/Physical_and_Theoretical_Chemistry_Textbook_M
aps/Supplemental_Modules_(Physical_and_Theoretical_Chemistry)/Physical_Properties_of_
Matter/Atomic_and_Molecular_Properties/Magnetic_Properties.
Flowers, Paul, Klaus Theopold, Richard Langley, and William R. Robinson. “Development of
Quantum Theory.” Chemistry 2e. OpenStax, October 19, 2018.
https://opentextbc.ca/chemistry2eopenstax/chapter/development-of-quantum-theory/.
Patalinghug, Wyona C, Vic Marie I Camacho, Fortunato B Sevilla, and Maria Cristina D Singson.
Teaching Guide for Senior High School General Chemistry 1. Diliman, Quezon City,
Philippines: Commission on Higher Education, 2016.
Pringle, Kenneth. “1.4: Electron Configuration and Orbital Diagrams.” Chemistry LibreTexts.
Libretexts, July 11, 2020.
https://chem.libretexts.org/Under_Construction/Purgatory/Essential_Chemistry_(Curriki)
/Unit_1:_Atomic_and_Molecular_Structure/1.4:_Electron_Configuration_and_Orbital_Diagr
ams.

Answer Key
Activity No. 1:
Is it Quantum
Condensed Draw the orbital diagram for the
paramagnetic Number of the
Element Electron subshell in the highest energy level
or Last Electron
Configuration containing the last electron
diamagnetic? n ℓ ml ms
Na [Ne]3s1 ↑ paramagnetic* 3 0 0 1/2
Ca [Ar]4s2 ↑↓ diamagnetic* 4 0 0 -1/2

Cl [Ne]3s23p5 ↑↓ ↑↓ ↑ paramagnetic* 3 1 1 1/2

Sc [Ar]4s23d1 ↑ paramagnetic* 3 2 -2 1/2

Zn [Ar]4s23d10 ↑↓ ↑↓ ↑↓ ↑↓ ↑↓ diamagnetic* 3 2 2 -1/2

O [He]2s22p4 ↑↓ ↑ ↑ paramagnetic* 2 1 -1 -1/2

*ground state

Activity No. 2: Answers are expected to vary.

Author: Ryan Paul M. Vales


School/Station: Tagbina National High School
Division: Surigao del Sur
email address: ryanpaul.vales@deped.gov.ph
WEEKLY LEARNING ACTIVITY SHEETS
General Chemistry 1, Grade 11, Quarter 2, Week 2

LEWIS STRUCTURES OF IONS

Name: ___________________________________________ Section: ____________________

Learning Objective:

 Draw the Lewis structure of ions (STEM_GC11CBIId-g-70)

Time Allotment: 2 hours

Specific Objectives:

After the lesson, the students will be able to:


1. draw the Lewis electron dot structure of atoms and ions
2. determine the charge of the ions based from their group number and valence electrons
3. identify elements that will form cations and anions based on their ionization energy
and electron affinity
4. value the importance of ions in the human body.

Key Concepts

Lewis Dot Structure

- Gilbert N. Lewis developed a system of representing valence electrons in an atom known


as Lewis electron-dot structure (LEDS) or simply Lewis Structure. A Lewis structure is
composed of a symbol of an element represents the nucleus and inner electrons
surrounded by dots which represent the valence electrons. Valence electrons are atoms
in the outermost shell used to form chemical bonds. A maximum of two dots are placed
in each of the four sides of an element.
- For representative elements, the group number corresponds to the number of valence
electrons.
 Place one dot at a time on the four sides (top, right, bottom, left) of the element
symbol.
 Keep adding dots, pairing the dots until all are used up.

Figure 1. Lewis Dot Symbols for Representative Elements

 The specific placement of dots is not important.

Author: Pamela Lou C. Suazo


School/Station: Unidad National High School
Division: Surigao del Sur Division
email address: pamelalou.suazo@deped.gov.ph
Lewis Dot Structure for Ions

- Ions are formed when there is unequal number of protons and electrons. When a neutral
atom loses electron, it becomes a cation. If an atom gains an electron it will become an
anion.

- Ionization Energy (IE) is the amount of energy required to remove an electron. Ionization
energy increases from left to right across a period. It decreases from top to bottom in a
group. This means that elements under Group 1A have the lowest ionization energy in every
period while elements under Group 8A have the highest ionization energy in every period.
Elements with low ionization energy such as metals have the tendency to give up electrons
easily forming cations with a positive (+) charge.

Figure 2. Ionization energy trend based on the periodic table.

- For a metal, the total number of dots is the maximum number of electrons an atom loses
to form a cation with a positive (+) charge.

- Steps in drawing the Lewis structures of ions:


Step 1. Determine the valence electrons of every atom.
Step 2. Determine how many electrons are lost or gained.
Assign a positive (+) charge for ion which loses electrons and negative (-)
charge for ion which gains electrons.

Example: Let’s consider Lithium atom.

Step 1. Determine the valence electrons of every atom.


Lithium is under group 1A (1 valence electron)
Step 2. Determine how many electrons are lost or gained.
Lithium is a metal that will lose one valence electron in order to become
stable.

1s22s1 1s2
[He] 2s1 [He]

Author: Pamela Lou C. Suazo


School/Station: Unidad National High School
Division: Surigao del Sur Division
email address: pamelalou.suazo@deped.gov.ph
- Electron Affinity (EA) is the energy released when an atom accepts an electron.
Electron affinity increases from left to right across a period. It decreases from top to
bottom in a group. This simply means that elements on the right side of the periodic
table which are mostly nonmetals have a higher tendency to accept electrons forming
anions with a negative (-) charge.

- Generally, ionization energy measures the energy change when an atom loses an
electron while electron affinity measures the energy change when an atom gains an
electron.

Figure 3. Electron affinity trend based on the periodic table.

- For a nonmetal, the number of unpaired dots is the number of electrons that become
paired either through electron gain or through electron sharing to form an anion with a
negative (-) charge.

Example: Let’s consider Nitrogen atom.


Step 1. Determine the valence electrons of every atom.
Nitrogen is under group 5A (5 valence electrons)
Step 2. Determine how many electrons are lost or gained.
Nitrogen is a nonmetal that will gain three valence electrons in order to
become stable.

1s22s22p3 1s22s22p6
[He] 2s22p3 [Ne]

- Atoms form bonds with other atoms in order to have a stable electronic configuration.
This will happen if atoms are isoelectronic with its nearest noble gas or having 8
electrons in its outermost shell, except for He which is stable with 2 valence electrons.

Author: Pamela Lou C. Suazo


School/Station: Unidad National High School
Division: Surigao del Sur Division
email address: pamelalou.suazo@deped.gov.ph
Activity No. 1 – Road to my Lewis

What you need:


Periodic Table

What to do: Complete the table with the information required.

Lewis Structure Number of Electrons


Group Valence Metal/
Element Neutral
Number Electrons Nonmetal Ion Gained Lost
Atom
Lithium 1A 1 Metal Li+ 1

Nitrogen 5A 5 Nonmetal 3

Cesium

Barium

Phosphorus

Sulfur

Bromine

Guide Questions
1. Which type of elements will likely to form a cation? Anion?
2. How are cations and anions formed?
3. What did you notice about the charge of the ion?

Author: Pamela Lou C. Suazo


School/Station: Unidad National High School
Division: Surigao del Sur Division
email address: pamelalou.suazo@deped.gov.ph
Activity No. 2 – Ions in the Body

What you need:


Periodic Table

What to do: Read the text below and identify the important ions present in our body.
Complete the table below by identifying of the ions, group number and valence electrons. In
the last column, draw the structures of these ions.

Important Ions in the Body

Ions are important in our body for their physiological and metabolic
functions. We can obtain these ions from the food that we eat. Salt and cheese
provide sodium ions which are important in regulating body fluids. Bananas,
orange juice, milk and potatoes are rich in potassium ions which also regulates
body fluids and cellular functions. Milk, yogurt, cheese, greens and spinach are
great sources of calcium ion which is a major cation present in our bone. These
ions should be present in balanced amount in order maintain stable internal
condition.

Group Valence
Ions in the Body Lewis Structure
Number Electrons
1.

2.

3.

Reflection

1. How are ions formed?


2. If you were an element, which group would you choose? Do you want to be a giver or a
receiver? Why?
3. What will happen if there is an imbalance of the important ions in our body?

RUBRICS
3 2 1 0
High level of Moderate level of Low level of science No explanation at
science concept science concept concept all.
understanding is understanding is understanding is
apparent in the apparent in the apparent
explanation with explanation with with
no misconceptions. minimal misconceptions.
misconceptions.

Author: Pamela Lou C. Suazo


School/Station: Unidad National High School
Division: Surigao del Sur Division
email address: pamelalou.suazo@deped.gov.ph
References for learners

Agnew, Marisa Alviar, and Henry Agnew. 2020. Chemistry LibreTexts. August 11. Accessed
October 29, 2020.
https://chem.libretexts.org/Courses/College_of_Marin/CHEM_114%3A_Introductory_
Chemistry/10%3A_Chemical_Bonding/10.03%3A_Lewis_Structures_of_Ionic_Compou
nds-_Electrons_Transferred.

Bayquen, Aristea V., and Gardee T. Peña. 2016. General Chemistry 1. Quezon City: Phoenix
Publishing House, Inc.

Brown, Theodore L., H. Eugene Lemay, Bruce E. Bursten, Catherine J. Murphy, Patrick M.
Woodward, and Mathew W. Stoltzfus. Chemistry the Central Science. 14th ed. New
York: Pearson, 2018.

Patalinghug, Wyona C., Vic Marie I. Camacho, Fortunato B. Sevilla III, and Maria Cristina
D. Singson. 2016. Teaching Guide for Senior High School General Chemistry 1. Quezon
City: Commission on Higher Education.

Timberlake, Karen C. Chemistry, an Introduction to General, Organic, and Biological


Chemistry, Twelfth Edition. Upper Saddle River: Prentice Hall, 2015.

Zumdahl, Steven S., and Susan A. Zumdahl. 2014. Chemistry. Belmont, CA: Brooks/Cole,
Cengage Learning.

Answer Key

Activity No. 1 – Road to my Lewis

Lewis Structure Number of Electrons


Group Valence Metal/
Element Neutral
Number Electrons Nonmetal Ion Gained Lost
Atom

Lithium 1A 1 Metal Li+ 1

Nitrogen 5A 5 Nonmetal 3

Cesium 1A 1 Metal Cs+ 1

Barium 2A 2 Metal Ba2+ 2

Phosphorus 5A 5 Nonmetal 3

Sulfur 6A 6 Nonmetal 2

Bromine 7A 7 Nonmetal 1

Author: Pamela Lou C. Suazo


School/Station: Unidad National High School
Division: Surigao del Sur Division
email address: pamelalou.suazo@deped.gov.ph
Guide Questions

1. Metals tend to lose electrons forming cations while nonmetals tend to gain electrons
forming into anions.
2. Cation is formed when a neutral atom loses one or more electrons.
Anion is formed when a neutral atom gains one or more electrons.
3. The charge of the ion corresponds to the number of electrons lost or gained.

Activity No. 2 – Ions in the Body

Group Valence
Ions in the Body Lewis Structure
Number Electrons
1. Sodium 1A 1 Na+

2. Potassium 1A 1 K+

3. Calcium 2A 2 Ca2+

Author: Pamela Lou C. Suazo


School/Station: Unidad National High School
Division: Surigao del Sur Division
email address: pamelalou.suazo@deped.gov.ph
WEEKLY LEARNING ACTIVITY SHEETS
General Chemistry 1, Grade 11, Quarter 2, Week 3

COVALENT COMPOUNDS

Name: ___________________________________________ Section: ____________________

Learning Objective:

 Apply the octet rule in the formation of molecular covalent compounds


(STEM_GC11CBIId-g-76)
 Write the formula of molecular compounds formed by the nonmetallic elements of
the representative block
(STEM_GC11CBIId-g-77)
 Draw Lewis structure of molecular covalent compounds
(STEM_GC11CBIId-g-78)

Time allotment: 4 hours

Specific Objectives:

After the lesson, the students will be able to:


1. illustrate how covalent compounds are formed
2. apply the octet rule in the formation of covalent compounds
3. draw the Lewis structure of covalent compounds
4. draw the resonance structures of covalent compounds
5. describe some exceptions of the octet rule
6. name covalent compounds given their formula and write formula given the name of
covalent compounds
7. identify covalent compounds that are used in everyday living

Key Concepts

- Covalent compounds are formed when a pair of electrons are shared between
nonmetals.
- Octet Rule states that when atoms gain, lose, or share electrons to form bonds
with other atoms they tend to achieve the electron configuration of the noble gas
nearest to them with eight valence electrons.
- There are three types of covalent bonds namely single bond, double bond and triple
bond.
- Bonding electrons are electrons shared between atoms.
- Nonbonding electrons are electrons that does not participating in bonding or not
shared between other atoms.
- Lewis structure shows how electrons are arranged in an atom.
- Steps in drawing the Lewis structures of compounds:
Step 1. Determine the valence electrons of every atom involved.
Step 2. Get the sum of the all the valence electrons of atoms.
Step 3. Use a pair of electrons to form a single bond between atoms.
Step 4. Arrange the remaining electrons around the atom to satisfy the octet rule
except for Hydrogen which is stable with 2 valence electrons.

Author: Pamela Lou C. Suazo


School/Station: Unidad National High School
Division: Surigao del Sur Division
email address: pamelalou.suazo@deped.gov.ph
Example: Draw the Lewis structure of F2

Step 1. Determine the valence electrons of every atom involved.


Fluorine from group 7 (7 valence electrons)

Step 2. Get the sum of the all the valence electrons of atoms.
Total valence electrons:
Fluorine = 7 valence electrons x 2 atoms
= 14 electrons

Step 3. Use a pair of electrons to form a single bond between atoms.

( ) single line bond is equivalent to 2 electrons


14 electrons – 2 electrons = 12 electrons

Step 4. Arrange the remaining electrons around the atom to satisfy the octet rule
except for Hydrogen which is stable with 2 valence electrons.

Lone pair

Bond pair

Check: Count the number of valence electrons whether each atom of Fluorine satisfies the
octet rule.
Fluorine = bonding electrons + nonbonding electrons
Fluorine = (1 x 2 e-) + 6 e-
Fluorine = 2 e- + 6 e-
Fluorine = 8 e-

Example: Draw the Lewis structure of CCl4

Step 1. Carbon from group 4A (4 valence electrons);


Chlorine from group 7A (7 valence electrons)

Step 2. Carbon = 4 valence electrons x 1 atom


= 4 electrons
Chlorine = 7 valence electrons x 4 atoms
= 28 electrons

Total valence electrons = 4 e-+28 e-


= 32 e-

Step 3. The least electronegative atom occupies the central position.

4 single line bonds x 2 e- = 8 e-


32 e- – 8 e- = 24 e-

Author: Pamela Lou C. Suazo


School/Station: Unidad National High School
Division: Surigao del Sur Division
email address: pamelalou.suazo@deped.gov.ph
Step 4.

Check: Count the number of valence electrons whether each atom satisfies the octet rule.

Carbon = bonding electrons + nonbonding electrons


Carbon = (4 x 2e-) + 0
Carbon = 8 e-

Chlorine = bonding electrons + nonbonding electrons


Chlorine = (1 x 2 e-) + 6 e-
Chlorine = 2 e - + 6 e-
Chlorine = 8 e-

Example: Draw the Lewis structure of CO2

Step 1. Carbon (4 valence electrons); Oxygen (6 valence electrons)

Step 2. Carbon = 4 valence electrons x 1 atom


= 4 electrons
Oxygen = 6 valence electrons x 2 atoms
= 12 electrons

Total valence electrons = 4 e- + 12 e-


= 16 e-

Step 3. The least electronegative occupies the central position.

2 single line bonds x 2 electrons = 4 electrons


16 electrons – 4 electrons = 12 electrons

Step 4.

Check: Count whether each atom satisfies the octet rule.

Oxygen (1) = 7 e-
Carbon = 6 e-
Oxygen (2) = 7 e-

The unpaired electrons of O will pair up with the unpaired electrons of C.

Author: Pamela Lou C. Suazo


School/Station: Unidad National High School
Division: Surigao del Sur Division
email address: pamelalou.suazo@deped.gov.ph
Check: Count the number of valence electrons whether each atom satisfies the octet rule.

Carbon = bonding electrons + nonbonding electrons


Carbon = 4 x 2e-
Carbon = 8e-

Oxygen = bonding electrons + nonbonding electrons


Oxygen = (2 x 2e-) + 4e-
Oxygen = 8e-

Example: Draw the Lewis structure N2.

Step 1. Nitrogen (5 valence electrons)

Step 2. Total valence electrons:


Nitrogen = 5 valence electrons x 2 atoms
= 10 electrons

Step 3.
10 electrons – 2 electrons = 8 electrons

Check: Count the number of valence electrons whether each atom satisfies the octet rule.

Nitrogen (1) = 6 electrons


Nitrogen (2) = 6 electrons

Pair up the unpaired electrons of Nitrogen

Check: Count the number of valence electrons whether each atom satisfies the octet rule.

Nitrogen = bonding electrons + nonbonding electrons


Nitrogen = (3 x 2e-) + 2 e-
Nitrogen = 8 e-

LEWIS STRUCTURE AND RESONANCE

1. Write the Lewis structure of ozone, O3.


a. Determine the valence electrons of every atom involved.
Oxygen from Group 6A (6 valence electrons)

b. Get the sum of the all the valence electrons of atoms.


Oxygen = 6 valence electrons x 3 atoms
= 18 electrons

Author: Pamela Lou C. Suazo


School/Station: Unidad National High School
Division: Surigao del Sur Division
email address: pamelalou.suazo@deped.gov.ph
c. Use a pair of electrons to form a single bond between atoms.

d. Arrange the remaining electrons around the atom to satisfy the octet rule.

or
Check: Count the number of valence electrons whether each atom satisfies the octet rule.

1 2 3

Oxygen (1) = bonding electrons + nonbonding electrons


= (2 x 2e-) + 2 e-
= 8e-

Oxygen (2) = bonding electrons + nonbonding electrons


= (3 x 2e-) + 2 e-
= 8e-

Oxygen (3) = bonding electrons + nonbonding electrons


= (1 x 2e-) + 6 e-
= 8e-

- Based from the Lewis structure of ozone above, there are two possible Lewis structures.
However, experimental results show that the bond length of ozone is between that of a
single bond and a double bond. This implies that we cannot use the two Lewis structures
above to represent ozone.
- In order to represent the Lewis structure of ozone accurately, a double-sided arrow is
placed in between the structures of ozone. These structures of ozone are called resonance
structures.


-
- A resonance structure is one of two or more Lewis structures for a molecule that cannot
be represented accurately by only one Lewis structure.
- What is the correct representation for ozone?
Answer: The two resonance structures shown with the double arrow.

EXCEPTIONS TO THE OCTET RULE


- The octet rule has limitations in many situations involving covalent bonding. These
exceptions to the octet rule are of three main types:
1. Molecules with fewer than an octet of valence electrons.
This type of exception occurs when there are fewer than eight valence electrons
around the central atom. An example of this case is BeH2, Beryllium Hydride.

There are only 4 electrons around Be atom.

2. Molecules containing an odd number of electrons.

Author: Pamela Lou C. Suazo


School/Station: Unidad National High School
Division: Surigao del Sur Division
email address: pamelalou.suazo@deped.gov.ph
Nitric oxide (NO) and nitrogen dioxide (NO2) are examples of molecules with odd
number of electrons which do not follow the octet rule.

and
3. Molecules with more than an octet of valence electrons.
Elements along the 3rd period and beyond have 3d orbitals that can participate in
chemical bonding. Hence, it is possible for the central atoms from 3rd period and
beyond to have expanded. Sulfur hexafluoride (SF6) and phosphorus pentachloride
(PCl5) are examples of compound having more than 8 electrons around the central
atom.
SF6

12 electrons around the


central atom, Sulfur

PCl5

10 electrons around the


central atom, Phosphorus

Guidelines in naming covalent compounds and writing their formulas:


 For binary compound, state the name of the first element while the name of the
second element end in -ide.
Example:
HCl - Hydrogen chloride
HF - Hydrogen fluoride
 Prefixes are added to the name of the element to denote the number of atoms
present. For the first element, the prefix “mono” is usually omitted. When the
prefix ends in “a” or “o” and the name of the second element begins with “a”
vowel, the “a” or “o” of the prefix is often dropped.

No. of atoms Prefix No. of atoms Prefix


1 mono- 6 hexa-
2 di- 7 hepta-
3 tri- 8 octa-
4 tetra- 9 nona-
5 penta- 10 deca-

Example 1: Write the name of the following covalent compounds.


NO2 - nitrogen dioxide
N2O4 - dinitrogen tetroxide
SF6 - sulfur hexafluoride

Example 2: Write the chemical formula of covalent compounds.


sulfur trioxide - SO3
diboron trioxide - B2O3
iodine pentafluoride- IF5

Author: Pamela Lou C. Suazo


School/Station: Unidad National High School
Division: Surigao del Sur Division
email address: pamelalou.suazo@deped.gov.ph
Activities

Activity 1. Draw Me!

What you need:


Periodic Table

What to do:
Draw the Lewis structure of the following covalent compounds.
1. O2
2. CH4
3. NCl3
4. HCN
5. HBr

Activity 2. What’s my formula!

What you need:


Periodic Table

What to do:
Write the chemical formula of the following molecular compounds.
1. Dichlorine hexoxide
2. Silicon dioxide
3. Dinitrogen trioxide
4. Sulfur pentoxide
5. Hydrogen sulfide

Activity 3. Designing my Own Brochure!

What you need:


Colored paper scissors glue

What to do:
1. Look for at least two examples of covalent compounds found in the
environment.
2. Write a brief explanation about the covalent compounds.
3. Write the chemical formula and name of the chemical compound.
4. Draw the structure of the compound.

Sample:

Author: Pamela Lou C. Suazo


School/Station: Unidad National High School
Division: Surigao del Sur Division
email address: pamelalou.suazo@deped.gov.ph
RUBRICS
3 2 1 0
All sentences
Most sentences
All sentences are are well
are well
well constructed constructed
Structure constructed with No explanation at
with no error in with several
(x1) minimal error in all.
grammar or errors in
grammar or
spelling. grammar or
spelling.
spelling
Content High level of
(x3) science concept Some of the
The information
is present. The information
presented are No explanation at
information presented are
accurate and all.
presented are inaccurate or
concise.
accurate and unclear.
concise.
Creativity Creative Creative Creativity is No creative
(x2) elements elements are lacking. Creative elements present.
enhance the present but do elements can
purpose of the not enhance the distract the
brochure. purpose of the purpose of the
brochure. brochure.

Assessment

1. What type of bond is formed when electrons are shared between atoms?
A. covalent bond.
B. chemical bond
C. ionic bond
D. metallic bond

2. How many electrons are shared between


atoms of carbon and hydrogen in methane?
A. 2
B. 4
C. 6
D. 8

3. Which of the following elements will have 5 electrons in its Lewis dot structure?
A. Boron
B. Carbon
C. Nitrogen
D. Sulfur

4. Dinitrogen monoxide is a greenhouse gas that enters the atmosphere principally


from natural fertilizer breakdown. What is the chemical formula of dinitrogen
monoxide?
A. NO
B. NO2
C. N2O
D. N2O2

Author: Pamela Lou C. Suazo


School/Station: Unidad National High School
Division: Surigao del Sur Division
email address: pamelalou.suazo@deped.gov.ph
5. What is the chemical formula of dichlorine heptoxide?
A. Cl2H6
B. Cl2H7
C. Cl2O6
D. Cl2O7

6. What is the chemical name of a compound which has two atoms of nitrogen and
five atoms of oxygen?
A. dinitrogen pentoxide
B. nitrogen dioxide
C. nitrogen monoxide
D. trinitrogen trioxide

7. Which is the correct Lewis structure for dihydrogen sulfide?

A. C.

B. D.

8. How many resonance structures can be drawn from O3?


A. 0
B. 2
C. 3
D. 4

9. Which of the following compounds have fewer than 8 valence electrons around
the central atom?
A. C2H6
B. AsF5
C. CFCl3
D. BF3

10. Which of the following is the correct Lewis structure for methanal, CH2O?

A. C.

B. D.

Author: Pamela Lou C. Suazo


School/Station: Unidad National High School
Division: Surigao del Sur Division
email address: pamelalou.suazo@deped.gov.ph
Reflection

1. Why are Lewis structures important?


2. An atom becomes stable when bonded with other atoms, how will you relate this in
human relationship?
3. What type of bond do you have with the people around you?

RUBRICS
3 2 1 0
High level of Moderate level of Low level of science No explanation at
science concept science concept concept all.
understanding is understanding is understanding is
apparent in the apparent in the apparent
explanation with explanation with with
no misconceptions. minimal misconceptions.
misconceptions.

References for learners

Bayquen, Aristea V., and Gardee T. Peña. 2016. General Chemistry 1. Quezon City:
Phoenix Publishing House, Inc.

Patalinghug, Wyona C., Vic Marie I. Camacho, Fortunato B. Sevilla III, and Maria
Cristina D. Singson. 2016. Teaching Guide for Senior High School General
Chemistry 1. Quezon City: Commission on Higher Education.

Answer Key

Activity 1 Assessment
1. A
1.
2. D
3. C
4. C
2. 5. D
6. A
7. A
3. 8. B
4. 9. D
10. A
5.

Activity 2
1. Cl2O6
2. SiO2
3. N2O3
4. SO5
5. H 2S

Author: Pamela Lou C. Suazo


School/Station: Unidad National High School
Division: Surigao del Sur Division
email address: pamelalou.suazo@deped.gov.ph
Author: Pamela Lou C. Suazo
School/Station: Unidad National High School
Division: Surigao del Sur Division
email address: pamelalou.suazo@deped.gov.ph
WEEKLY LEARNING ACTIVITY SHEETS
General Chemistry 1 Grade 11, Quarter 2, Week 4

MOLECULAR GEOMETRY AND POLARITY

Name: ___________________________________________ Section: ____________________

Learning Objective:

 Describe the geometry of simple compounds (STEM_GC11CB-IId-g-81)

 Determine the polarity of simple molecules (STEM_GC11CB-IId-g-82)

Time allotment: 4 hours

Specific Objectives:
After the lesson, the students will be able to:
1. determine whether of a bond is ionic, nonpolar or polar covalent based on the
electronegativity difference
2. Apply the Valence Shell Electron Pair Repulsion Theory to predict the geometry of
molecules.
3. Predict the polarity of molecule

Key Concepts

Molecular Geometry

- Molecular geometry refers to the three-dimensional arrangement of atoms within a


molecule.
- The shape of a molecule is determined by its bond angles, the angles made by the lines
joining the nuclei of the atoms in the molecule. The bond angles of a molecule,
together with the bond lengths, define the shape and size of the molecule.
- The geometry of a molecule can be predicted using the assumption that electrons in
the valence shell around the central atom will repel from each other. Every electron
group around a central atom is located as far away as possible to minimize repulsions.
This is also known as Valence Shell Electron Pair Repulsion Theory (VSEPR).
- Key ideas in predicting molecular geometry using the VSEPR Theory
 Electron groups (one or more electron pairs) are located as far away from each
other as possible in order to minimize repulsion.
 Molecular geometry is determined by identifying the bonded pairs and lone
pairs around the central atom. Thus, it is important to know first the Lewis
structure of the compound.
 Single bonds, double bonds and triple bonds are considered as one electron
group.
 Lone pairs (LP) of electron occupy more volume than bond pairs (BP).
 LP-LP>LP-BP>BP-BP. Lone pair-lone pair repulsions are greater than lone pair-
bond pair repulsions and greater than bond pair-bond pair repulsion.

Author: Pamela Lou C. Suazo


School/Station: Unidad National High School
Division: Surigao del Sur Division
email address: pamelalou.suazo@deped.gov.ph
Atoms
Electron- Bonded Lone Three
Electron Bond Molecular
Group to Pairs of Example Dimensional
Groups Angle Geometry
Geometry Central Electrons Model
Atom
2 Linear 2 0 180° Linear CO2

Trigonal Trigonal
3 3 0 120° BF3
Planar Planar

Trigonal
3 2 1 <120° Bent NO2-
Planar

4 Tetrahedral 4 0 109.5° Tetrahedral CH4

Trigonal
4 Tetrahedral 3 1 <109.5° NH3
Pyramidal

4 Tetrahedral 2 2 <109.5° Bent H2O

90°
Trigonal (axial) Trigonal
5 5 0 PCl5
Bipyramidal 120° Bipyramidal
(equatorial)
90°
Trigonal (axial)
5 Bipyramidal 4 1 120° Seesaw SF4
(equatorial)

Trigonal
<90° (axial)
5 Bipyramidal 3 2 T-shaped ClF3

Trigonal
180°
5 Bipyramidal 2 3 Linear XeF2

Octahedral 90°
6 6 0 Octahedral SF6

Octahedral <90° Square


6 5 1 BrF5
pyramidal

Octahedral 90° Square


6 4 2 XeF4
planar

Table 1. Molecular Shapes for Central Atom with Two, Three and Four Bonded Atoms

Image source:
Theodore L. Brown et al., Chemistry the Central Science, 14th ed. (New York: Pearson, 2018), 389-392.
Example: Predict the molecular geometry of BeCl2.

Author: Pamela Lou C. Suazo


School/Station: Unidad National High School
Division: Surigao del Sur Division
email address: pamelalou.suazo@deped.gov.ph
a. Draw the Lewis structure of the molecule.
b. How many electron groups are present? 2 electron groups
c. How many bond pair/s are present around the central atom? 2 bond pairs
d. How many lone pair/s are present around the central atom? 0 lone pair
e. How will these electron groups orient themselves in order to minimize repulsion?
The combination between the bonded atoms and lone pairs, 2-0 will give BeCl2 a linear
electron group geometry and a linear shape.
f. What is the bond angle in Cl-Be-Cl? 180°

Example: Predict the molecular geometry of BCl3.

a. Draw the Lewis structure of the molecule.

b. How many electron groups are present? 3 electron groups


c. How many bond pairs are present around the central atom? 3 bond pairs
d. How many lone pair/s are present around the central atom? 0 lone pair
e. How will these electron groups orient themselves in order to minimize repulsion? The
combination between the bonded atoms and lone pairs, 3-0 will give BCl3 a trigonal
planar electron group geometry and a trigonal planar shape.
f. What is the bond angle in Cl-B-Cl? 120°

Example: Predict the molecular geometry of SO2.

A. Draw the Lewis structure of the molecule.

B. How many electron groups are present? 3 electron groups


C. How many bond pairs are present around the central atom? 2 bond pairs
D. How many lone pair/s are present around the central atom? 1 lone pair
E. How will these electron groups orient themselves in order to minimize repulsion?
The combination between the bonded atoms and lone pairs, 2-1 will give SO2 a trigonal
planar electron group geometry but the molecular geometry is bent shape.
F. What is the bond angle in O-S-O? 120°

- The table above will guide you how to identify the electron group geometry as well as
the molecular geometry of compounds. Note that, if there are no lone pairs around the
central atom the electron group geometry is the same with its molecular geometry.

Example: Predict the molecular geometry of acetone in every carbon atom.

Author: Pamela Lou C. Suazo


School/Station: Unidad National High School
Division: Surigao del Sur Division
email address: pamelalou.suazo@deped.gov.ph
Electron groups 4 3 4
Bond pairs 4 3 4
Lone pairs 0 0 0
Electron group
Tetrahedral Trigonal planar Tetrahedral
geometry
Molecular geometry Tetrahedral Trigonal planar Tetrahedral
Bond angle 109.5 ° 120 ° 109.5 °

Bond Polarity
- Covalent bonds may be classified into nonpolar or polar covalent bonds depending on
the electronegativity difference between atoms.
- Electronegativity refers to the ability of an atom to draw or attract the electrons of
other atoms toward itself. Elements with greater electronegativity value such as
Fluorine has a higher tendency to attract the electrons of other atoms.
- In a polar covalent bond, the shared electrons are more attracted to the more
electronegative atom making it partially negative (ẟ-) while the other end becomes
partially positive (ẟ+) due to the lack of electrons in an atom. Polar covalent bond that
has a separation of charges is called a dipole. An arrow ( ) is used to represent
dipole where the arrow head is pointed towards the more electronegative atom.
- In a nonpolar covalent bond, electrons are shared equally between atoms. The
electronegativity difference ranges from 0.0 to 0.4.

Bond Electronegativity Difference


(∆EN)
Ionic Bond ∆EN ≥ 1.8
Polar Covalent Bond 1.8 > ∆EN > 0.4
Nonpolar Covalent Bond ∆EN ≤ 0.4

Figure 1. Electronegativity Difference and Type of Bonds


Image source:
Karen C. Timberlake, Chemistry, an Introduction to General, Organic, and Biological
Chemistry, Twelfth Edition (Upper Saddle River: Prentice Hall, 2015), 190.

Author: Pamela Lou C. Suazo


School/Station: Unidad National High School
Division: Surigao del Sur Division
email address: pamelalou.suazo@deped.gov.ph
- Determine the type of bond between each pair of atoms.
Use the electronegativity values provided in Figure 3.

Electronegativity
Type of Bond Reason
Difference

Ca─Cl
3.0-1.0=2.0 Ionic bond Greater than 1.8
bond in CaCl2
C─S Nonpolar Between 0 and
2.5-2.5=0.0
bond in CS2 covalent bond 0.4

H─Cl Polar covalent Greater than 0.4


3.0-2.1=0.9
bond in HCl Bond but less than 1.8

Dipole Moments and Polarity of Molecule

- If the electronegativity difference is greater than 1.8, electrons are not shared but
transferred from one atom to another which results to formation of an ionic bond.
- Consider the atoms of hydrogen and chlorine. Hydrogen atom has an electronegativity
value of 2.1 and chlorine atom has 3.0. These values imply that the electrons of
hydrogen are drawn towards chlorine atom which results to a partially negative
chlorine and partially positive hydrogen.

H─Cl
ẟ+ ẟ-
- Between carbon and sulfur, both atoms have the same electronegativity value of 2.5.
This means that the electrons are shared equally between carbon and sulfur making
the bond nonpolar.

Polarity of Simple Compounds


- Polar molecules always contain one end that is more negatively charged. Polarity
occurs when the dipoles from the individual bond do not cancel each other.

Example 1. Is water a polar molecule or not?


Electronegativity:
O = 3.5 ; H = 2.1
ẟ-

ẟ+ ẟ+

Electronegativity difference = 3.5 – 2.1 = 1.4 (Polar covalent bond)

Consider the bent shape of water. The individual dipoles are directed towards oxygen,
therefore, they do not cancel each other which explains that water molecule is polar.
The net dipole moment of oxygen is upward.

Author: Pamela Lou C. Suazo


School/Station: Unidad National High School
Division: Surigao del Sur Division
email address: pamelalou.suazo@deped.gov.ph
- Nonpolar molecules occur when the polar bonds cancel each other because of its
asymmetrical arrangement. Some polar molecules have polar bonds but the resulting
molecule is nonpolar.

- Example 2. Is CO2 a polar molecule or not?


Electronegativity:
C = 2.5 ; O = 3.5

ẟ- ẟ+ ẟ-

Electronegativity difference = 3.5 – 2.5 = 1.0


Net dipole moment = 1.0 – 1.0 = 0.0 (Nonpolar covalent bond)

The bonds between carbon and oxygen are both polar. However, these bonds are
directed in opposite direction which cancels out resulting to a net dipole moment of 0.
- The polarity of substances will help us in determining its solubility in different
solvents. Solubility of substances follows the general rule “like dissolves like”. Polar
solute dissolves in a polar solvent. Nonpolar solute dissolves in a nonpolar solvent

Activity 1. Complete me.

What you need:


Periodic Table
What to do:
Complete the table with the needed information.

Lewis Electron Electron- Bonded Lone Bond Molecular


Structure Groups Group Atoms Pairs Angle Geometry
Geometry
BeCl2 2 linear 2 0 180° Linear
1. HCN

2. NCl3

3. CH4

4. PCl5

Author: Pamela Lou C. Suazo


School/Station: Unidad National High School
Division: Surigao del Sur Division
email address: pamelalou.suazo@deped.gov.ph
Activity 2. Identify our bond.

What you need:


Electronegativity Values

Image source:
Karen C. Timberlake, Chemistry, an Introduction to General, Organic, and
Biological Chemistry, Twelfth Edition (Upper Saddle River: Prentice Hall, 2015),
189.

Figure 3. Electronegativity Values

What to do: Identify the type of bond between each pair of atoms. Use the electronegativity
values provided in Figure 3.
Electronegativity
Type of Bond
Difference
C─O
3.5-2.5=1.0 Polar Covalent Bond
bond in CO
1. H─H
bond in H2
2. Br─Cl
bond in BrCl3
3. H─Br
bond in HBr
4. As─Cl
bond in AsCl5
5. P─Br
bond in PBr5

Author: Pamela Lou C. Suazo


School/Station: Unidad National High School
Division: Surigao del Sur Division
email address: pamelalou.suazo@deped.gov.ph
Activity 3. Polar or Nonpolar?

What you need:


Periodic Table

What to do: Determine the molecular geometry and polarity of the following molecules.

Lewis Molecular
Polarity
Structure Geometry
1.

2.

3.
4.

5.

Assessment

Choose the letter of the best answer.

1. How many electron groups are present around


the central atom carbon in CH2O?

A. 1
B. 2
C. 3
D. 4

2. How many electrons does a double covalent bond represent?


A. 2
B. 4
C. 6
D. 8

3. Which statement best explains why CH2O is more polar than CCl2O?

CCl2O CH2O

A. because of the relatively upward dipoles in it that will all add up


B. because hydrogen has greater electronegativity value than Chlorine.
C. because it is surrounded by hydrogen atom that is directed downward

Author: Pamela Lou C. Suazo


School/Station: Unidad National High School
Division: Surigao del Sur Division
email address: pamelalou.suazo@deped.gov.ph
D. because of its opposing dipoles which decrease the overall dipole moment of the
molecule

4. Which pair of atoms is the most polar?


Electronegativity Difference
A. C—O 3.5 – 2.5 = 1.0
B. C—S 2.5 – 2.5 = 0
C. Se—P 2.4 – 2.1 = 0.3
D. Ca—Cl 3.0 – 1.0 = 2.0

5. What is the molecular geometry of CS2?


A. linear
B. octahedral
C. tetrahedral
D. trigonal planar

6. Which of the following has tetrahedral electron domain geometry?


A. HBr
B. HCN
C. O3
D. OF2

7. Which of the following has a trigonal planar molecular geometry?

A. C.

B. D.

8. Which of the following is a nonpolar covalent bond?


A. CO
B. NF3
C. Al2O3
D. SF6

9. Which of the following illustrations is correct?

A. H─F C. S─Cl

B. O─S D. B─H

Author: Pamela Lou C. Suazo


School/Station: Unidad National High School
Division: Surigao del Sur Division
email address: pamelalou.suazo@deped.gov.ph
10. The molecule boron trifluoride, BF3, assumes a trigonal planar geometry with boron
as the central atom. Which statement correctly describes the polarity of the B-F
bond and the polarity of the molecule?
A. The B-F bond is polar; the BF3 molecule is polar.
B. The B-F bond is nonpolar; the BF3 molecule is polar.
C. The B-F bond is polar; the BF3 molecule is nonpolar.
D. The B-F bond is nonpolar; the BF3 molecule is nonpolar.

Reflection

1. What would be the consequences if water becomes a nonpolar molecule?


2. What type of covalent bond do you have with your siblings or parents, is it polar or
nonpolar? Why?

RUBRICS
3 2 1 0
High level of Moderate level of Low level of science No explanation at
science concept science concept concept all.
understanding is understanding is understanding is
apparent in the apparent in the apparent
explanation with explanation with with
no misconceptions. minimal misconceptions.
misconceptions.

References for learners

Bayquen, Aristea V., and Gardee T. Peña. 2016. General Chemistry 1. Quezon City: Phoenix
Publishing House, Inc.

Brown, Theodore L., H. Eugene Lemay, Bruce E. Bursten, Catherine J. Murphy, Patrick M.
Woodward, and Mathew W. Stoltzfus. Chemistry the Central Science. 14th ed. New
York: Pearson, 2018.

Patalinghug, Wyona C., Vic Marie I. Camacho, Fortunato B. Sevilla III, and Maria Cristina
D. Singson. 2016. Teaching Guide for Senior High School General Chemistry 1.
Quezon City: Commission on Higher Education.

Timberlake, Karen C. Chemistry, an Introduction to General, Organic, and Biological Chemistry,


Twelfth Edition. Upper Saddle River: Prentice Hall, 2015.

Author: Pamela Lou C. Suazo


School/Station: Unidad National High School
Division: Surigao del Sur Division
email address: pamelalou.suazo@deped.gov.ph
Answer Key

Activity 1.

Electron-
Lewis Electron Bonded Lone Bond Molecular
Compound Group
Structure Groups Atoms Pairs Angle Geometry
Geometry
BeCl2 2 Linear 2 0 180° Linear
1. HCN 2 Linear 2 0 180° Linear
Trigonal
2. NCl3 4 Tetrahedral 3 1 109.5°
pyramidal

3. CH4 4 Tetrahedral 4 0 109.5° Tetrahedral

Trigonal 120° Trigonal


4. PCl5 5 5 0
bipyramidal 90° bipyramidal

Activity 2.

Electronegativity
Type of Bond
Difference
1. H─H
2.1-2.1 = 0.0 Nonpolar covalent bond
bond in H2
2. Br─Cl
3.0-2.8 = 0.2 Nonpolar covalent bond
bond in BrCl3
3. H─Br
2.8-2.1 = 0.7 Polar covalent bond
bond in HBr
4. As─Cl
3.0-2.0 = 1.0 Polar covalent bond
bond in AsCl5
5. P─Br
2.8-2.1 = 0.7 Polar covalent bond
bond in PBr5

Activity 3.

Lewis Molecular
Polarity
Structure Geometry
1. Linear Polar

2. Linear Polar

Trigonal pyramidal Polar


3.
Linear Nonpolar
4.

Tetrahedral Nonpolar
5.

Author: Pamela Lou C. Suazo


School/Station: Unidad National High School
Division: Surigao del Sur Division
email address: pamelalou.suazo@deped.gov.ph
Assessment

1. C
2. B
3. A
4. A
5. A
6. D
7. A
8. D
9. C
10. C

Author: Pamela Lou C. Suazo


School/Station: Unidad National High School
Division: Surigao del Sur Division
email address: pamelalou.suazo@deped.gov.ph
1|Page
WEEKLY LEARNING ACTIVITY SHEETS
General Chemistry 1 Grade 11, Quarter 2, Week 5

ORGANIC COMPOUNDS

Families of Carbon Compounds: Functional Groups

Name: ___________________________________________ Section: ________________________________

Learning Competency:

Describe the different functional groups (STEM_GC11OCIIg-j-87)

Specific Objectives:

1. Describe the major functional groups in organic chemistry and name organic compounds;
2. Recognize from the structural formula which class of organic compounds a certain molecule
belongs; and
3. Identify all the functional groups present in a molecule given its structural formula.

Time allotment: 4 Hours

Key Concepts

 Organic chemistry is the study of the chemistry of carbon-containing compounds.

 Organic compounds are compounds containing one or more atoms of carbon EXCEPT oxides
of carbon (e.g., CO and CO2); carbides (e.g., CaC2, commonly known as “kalburo”); cyanides
(e.g., NaCN); carbonates (e.g., CaCO3); hydrogen carbonates or bicarbonates (e.g., NaHCO3); and
allotropes of carbon (e.g., diamond, graphite, graphene, and fullerenes such as
buckminsterfullerene, or C60).
 Carbon can form four covalent bonds. It forms strong bonds with a variety of elements,
especially H, O, N, and the halogens. Carbon also has an exceptional ability to bond to another
carbon atom/s forming a variety of molecules made up of chains (straight/unbranched,
branched, or cyclic) of carbon atoms in endless arrays.

 Molecular formula is a chemical formula that indicates the actual numbers of atoms present
in a molecule. For example, the molecular formula CH4 for methane tells us that methane has
one carbon atom and four hydrogen atoms.

 Structural formula displays the atoms of the molecule in the order they are bonded. It also
depicts how the atoms are bonded to one another. There are four types of structural formulas:
electron-dot formula, dash formula, condensed formula, and bond-line formula or skeletal formula.
Examples of these four types are shown below using 1-propanol as an example.

FIGURE 1: Ball-and-stick model and structural formulas for 1-propanol

 Although electron-dot formulas account explicitly for all the valence electrons in a molecule,
they are tedious and time-consuming to write. Dash, condensed, and bond-line formulas are
therefore used more often.

Author: CRISTOBERT C. AYATON


School/Station: LIANGA NATIONAL COMPREHENSIVE HIGH SCHOOL – LIANGA I DISTRICT
Division: SURIGAO DEL SUR
email address: cristobert.ayaton@deped.gov.ph
2|Page
A. Dash Structural Formula

 Dash formulas have lines that show bonding electron pairs and include elemental symbols for
all the atoms in a molecule.
 If we look at Fig.1b-d, we find that the chain of atoms in those formulas is “straight.” Fig.1a
and Fig.1e correspond more accurately to the actual shape of the molecule where the chain
of atoms is “not at all straight.” A better description is unbranched.
 Atoms joined by single bonds can rotate relatively freely with respect to one another.
 The relatively free rotation of single bonds mean that the chain of atoms can assume a variety
of arrangements. Consider the dash formulas of 1-propanol.

Equivalent dash formulas for 1-propanol

 Consider the compound called 2-propanol (or isopropyl alcohol), whose dash formula can be
written in a variety of ways:

Equivalent dash formulas for 2-propanol

B. Condensed Structural Formula

 In condensed formulas, all hydrogens that are attached to a particular carbon are usually
written immediately after that carbon. In fully condensed structural formulas, all of the atoms
that are attached to a particular carbon are usually written immediately after that carbon,
listing hydrogens first. Consider 2-chlorobutane as an example.

 The condensed formulas for 2-propanol can be written in four different ways:

Author: CRISTOBERT C. AYATON


School/Station: LIANGA NATIONAL COMPREHENSIVE HIGH SCHOOL – LIANGA I DISTRICT
Division: SURIGAO DEL SUR
email address: cristobert.ayaton@deped.gov.ph
3|Page
C. Bond-line Structural Formulas

 The most common type of structural formula used in organic chemistry, and the fastest to
draw, is the bond-line formula. Another term for bond-line formula is skeletal formula.
 Bond-line formula lacks symbols that are explicitly shown in the dash and condensed
formulas. It allows us to more quickly interpret molecular connectivity and compare one
molecular formula with another.
 In drawing bond-line formulas, we apply the following rules:

 Each line represents a bond.

 Each bend in a line and terminus of a line represents a carbon atom, unless another group
is shown explicitly.

 No Cs are written for carbon atoms, except optionally for CH 3 groups at the end of a chain
or branch.
 No Hs are shown for hydrogen atoms, unless they are needed to give a three-dimensional
perspective.
 The number of hydrogen atoms bonded to each carbon is inferred by assuming that as
many hydrogen atoms are present as needed to fill the valence shell of the carbon, unless
a charge is indicated.
 When an atom other than carbon or hydrogen is present, the symbol for that element is
written at the appropriate location (i.e., in place of a bend or at the terminus of the line
leading to the atom).

 Hydrogen atoms bonded to atoms other than carbon (e.g., oxygen or nitrogen) are written
explicitly such as the H in OH.

 Bond-line formulas are easy to draw for molecules with multiple bonds and for cyclic
molecules, as well. The following are some examples.

Author: CRISTOBERT C. AYATON


School/Station: LIANGA NATIONAL COMPREHENSIVE HIGH SCHOOL – LIANGA I DISTRICT
Division: SURIGAO DEL SUR
email address: cristobert.ayaton@deped.gov.ph
4|Page

SOLVED PROBLEM 1: Write the bond-line formula for CH3CH(CH3)CH2CH2CH2OH.

Strategy and Answer: We outline the carbon skeleton including the OH group as follows:

Then the bond-line formula is written as

 Hydrocarbons are compounds that contain only carbon and hydrogen atoms. Hydrocarbons
are further classified into aliphatic hydrocarbons (those that do not contain benzene ring) and
aromatic hydrocarbons (those that contain benzene ring). Names of hydrocarbons end in -ane,
-ene, or -yne which then tell us what kinds of carbon–carbon bonds are present. The name of
the parent compound is determined by the number of carbon atoms in the longest chain.

Table 1. Prefixes Used in Naming Hydrocarbons


Number of Carbon Atoms Prefix Number of Carbon Atoms Prefix
1 Meth- 6 Hex-
2 Eth- 7 Hept-
3 Prop- 8 Oct-
4 But- 9 Non-
2 Pent- 10 Dec-

A. Alkanes contain only carbon-carbon single bond (C―C), and this is indicated in the family
name and in names for specific compounds by an -ane ending.

 Alkanes, except cycloalkanes, have the general formula CnH2n+2 where n=1, 2, 3, ….
 They are also known as saturated hydrocarbons because these compounds contain the
maximum number of hydrogen atoms that can bond to carbon atoms present; that is, they
are saturated with hydrogen atoms.

Author: CRISTOBERT C. AYATON


School/Station: LIANGA NATIONAL COMPREHENSIVE HIGH SCHOOL – LIANGA I DISTRICT
Division: SURIGAO DEL SUR
email address: cristobert.ayaton@deped.gov.ph
5|Page
 Examples are methane (the simplest alkane), ethane, propane, and butane.
 The structural formulas for butane are as follows:

Dash formula Condensed formula Bond-line formula

Alkyl Groups and the Symbol R

 Alkyl groups are the groups that would be obtained by removing a hydrogen atom from an
alkane. These groups are identified for the purpose of naming compounds. These alkyl groups
have names that end in -yl.
 When the alkane is unbranched, and the hydrogen atom that is removed is a terminal
hydrogen atom, the names are straight forward. Examples of unbranched alkyl groups are
methyl, ethyl, propyl and butyl.

 Methyl group can be derived from methane by removing one H atom. Ethyl group can be
derived from ethane after the removal of one H atom attached from any of the two C atoms.
 For alkanes with more than two carbon atoms, more than one alkyl groups can be derived.
Two alkyl groups can be derived from propane. (1) Removal of one terminal H atom forms an
unbranched alkyl called propyl group; (2) removal of one H atom from the middle C atom
forms a branched alkyl called 1-methylethyl (systematic name) or isopropyl group (common
name).

Three-carbon groups

 There are four alkyl groups can be derived from an alkane with 4 carbon atoms.

Author: CRISTOBERT C. AYATON


School/Station: LIANGA NATIONAL COMPREHENSIVE HIGH SCHOOL – LIANGA I DISTRICT
Division: SURIGAO DEL SUR
email address: cristobert.ayaton@deped.gov.ph
6|Page

Four-carbon groups

 There is one of the seven five-carbon groups with an IUPAC approved common name: the 2,2-
dimethylpropyl group, commonly called the neopentyl group.

 Generally, the symbol R is used to represent any alkyl group. For example, R might be
methyl, ethyl, propyl, isopropyl, butyl, tert-butyl, etc.

Alkyl Halides or Haloalkanes

 Alkyl halides or haloalkanes are compounds in which halogens (fluorine, chlorine, bromine,
or iodine) replaces a hydrogen atom of an alkane.

 Alkyl halides or haloalkanes have the general formula RX where X = F, Cl, Br, or I. In naming
compounds with halogen substituents, we use fluoro, chloro, bromo, and iodo.

 Alkyl halides are classified as being primary (1°), secondary (2°), or tertiary (3°). (We do not
say first degree, second degree, or third degree, but rather we say primary, secondary, or
tertiary.) This classification is based on the carbon atom to which the halogen is
directly attached.

A. 1° Alkyl halide – If the carbon atom that bears the halogen is directly attached to only one
other carbon, the carbon atom is said to be a primary carbon and the alkyl halide is classified
as a primary alkyl halide.

B. 2° Alkyl halide – If the carbon that bears the halogen is itself directly attached to two
other carbon atoms, then the carbon is a secondary carbon and the alkyl halide is a
secondary alkyl halide.

C. 3° Alkyl halide - If the carbon that bears the halogen is directly attached to three other
carbon atoms, then the carbon is a tertiary carbon and the alkyl halide is a tertiary alkyl
halide.

Author: CRISTOBERT C. AYATON


School/Station: LIANGA NATIONAL COMPREHENSIVE HIGH SCHOOL – LIANGA I DISTRICT
Division: SURIGAO DEL SUR
email address: cristobert.ayaton@deped.gov.ph
7|Page

Nomenclature of Alkanes

 Today, chemists use a systematic nomenclature developed and updated by the International
Union of Pure and Applied Chemistry (IUPAC). Underlying the IUPAC system is a fundamental
principle: each different compound should have a different and unambiguous name.

 In IUPAC substitutive nomenclature a name may have as many as four features: locants,
prefixes, parent compound, and suffixes. Consider the following example:

The locant 4- tells that the substituent methyl group, named as a prefix, is attached to the
parent compound at C4. The parent compound contains six carbon atoms and no multiple
bonds, hence the parent name is hexane, and it is an alcohol; therefore it has the suffix -ol.
The locant 1- tells that C1 bears the hydroxyl group. In general, numbering of the chain
always begins at the end nearer the group named as a suffix.

The locant for a suffix (whether it is for an alcohol or another functional group) may be placed
before the parent name as in the above example or, according to a 1993 IUPAC revision of
the rules, immediately before the suffix. Both methods are IUPAC approved. Therefore, the
above compound could also be named 4-methylhexan-1-ol.

 Naming unbranched alkane is very straight forward using the prefix that corresponds to the
number of carbon atoms and an ending -ane. The names for the first twenty unbranched
alkanes are listed in Table 2.

Table 2. The First Twenty of the Unbranched Alkane Series

Author: CRISTOBERT C. AYATON


School/Station: LIANGA NATIONAL COMPREHENSIVE HIGH SCHOOL – LIANGA I DISTRICT
Division: SURIGAO DEL SUR
email address: cristobert.ayaton@deped.gov.ph
8|Page
 Branched alkanes are named according to the following rules:

1. Locate the longest continuous chain of carbon atoms; this chain determines the parent
name for the alkane. We designate the following compound, for example, as a hexane because
the longest continuous chain contains six carbon atoms:

The longest continuous chain may not always be obvious from the way the formula is written.
Notice, for example, that the following alkane is designated as a heptane because the longest
chain contains seven carbon atoms:

2. Number the longest chain beginning with the end of the chain nearer the substituent.
Applying this rule, we number the two alkanes that we illustrated previously in the following
way:

3. Use the numbers obtained by application of rule 2 to designate the location of the
substituent group. The parent name is placed last, and the substituent group, preceded by the
number designating its location on the chain, is placed first. Numbers are separated from words
by a hyphen. Our two examples are 2-methylhexane and 3-methylheptane, respectively:

4. When two or more substituents are present, give each substituent a number
corresponding to its location on the longest chain. For example, we designate the following
compound as 4-ethyl-2-methylhexane:

The substituent groups should be listed alphabetically (i.e., ethyl before methyl). In deciding on
alphabetical order, disregard multiplying prefixes such as “di” and “tri.”

5. When two substituents are present on the same carbon atom, use that number twice:

6. When two or more substituents are identical, indicate this by the use of the prefixes
di-, tri-, tetra-, and so on. Then make certain that each and every substituent has a number.
Commas are used to separate numbers from each other:

Author: CRISTOBERT C. AYATON


School/Station: LIANGA NATIONAL COMPREHENSIVE HIGH SCHOOL – LIANGA I DISTRICT
Division: SURIGAO DEL SUR
email address: cristobert.ayaton@deped.gov.ph
9|Page

Application of these six rules allows us to name most of the alkanes that we shall encounter.
Two other rules, however, may be required occasionally:

7. When two chains of equal length compete for selection as the parent chain, choose the
chain with the greater number of substituents:

8. When branching first occurs at an equal distance from either end of the longest chain,
choose the name that gives the lower number at the first point of difference:

SOLVED PROBLEM 1: Provide an IUPAC name for the alkane

Strategy and Answer: The longest chain (shown in blue) contains seven carbon atoms;
therefore, the parent name is heptane. There are two methyl substituents (shown in red). We
number the carbon atoms so as to give the first methyl group a lower number. The correct name,
therefore, is 3,4-dimethylheptane. Numbering the chain from the other end to give 4,5-
dimethylheptane would have been incorrect.

SOLVED PROBLEM 2: Give the systematic name for the following alkane

Strategy and Answer: We are given with the condensed formula of the alkane. The longest
continuous chain extends from the upper left CH3 group to the lower left CH3 group and is seven
C atoms long:

Author: CRISTOBERT C. AYATON


School/Station: LIANGA NATIONAL COMPREHENSIVE HIGH SCHOOL – LIANGA I DISTRICT
Division: SURIGAO DEL SUR
email address: cristobert.ayaton@deped.gov.ph
10|Page
The parent chain is heptane, and two methyl substituents are branching off the main chain.
The exact location of the two methyl substituents are in C3 and C4. Therefore, the name of the
alkane is 3,4-dimethylheptane. This is the same molecule as in SOLVED PROBLEM 1. They
may look different at first glance, but they are actually the same.

SOLVED PROBLEM 3: Which structure does not represent 2-methylpentane?

Answer: Molecules (a), (b), and (c) represent 2-methylpentane while molecule (d) represent 3-
methylpentane.

Nomenclature of Cycloalkanes

 Cycloalkanes are alkanes that form rings or cycles. The general formula is CnH2n.

 Cycloalkanes are named by adding “cyclo” before the parent name.

1. Cycloalkanes with one ring and no substituents: Count the number of carbon atoms in
the ring, then add “cyclo” to the beginning of the name of the alkane with that number of
carbons. For example, cyclopropane has three carbons and cyclopentane has five carbons.

2. Cycloalkanes with one ring and one substituent: Add the name of the substituent to the
beginning of the parent name. For example, cyclohexane with an attached isopropyl group is
isopropylcyclohexane. For compounds with only one substituent, it is not necessary to specify
a number (locant) for the carbon bearing the substituent.

3. Cycloalkanes with one ring and two or more substituents: For a ring with two
substituents, begin by numbering the carbons in the ring, starting at the carbon with the
substituent that is first in the alphabet and number in the direction that gives the next

Author: CRISTOBERT C. AYATON


School/Station: LIANGA NATIONAL COMPREHENSIVE HIGH SCHOOL – LIANGA I DISTRICT
Division: SURIGAO DEL SUR
email address: cristobert.ayaton@deped.gov.ph
11|Page
substituent the lower number possible. When there are three or more substituents, begin at the
substituent that leads to the lowest set of numbers (locants). The substituents are listed in
alphabetical order, not according to the number of their carbon atom.

4. When a single ring system is attached to a single chain with a greater number of carbon
atoms, or when more than one ring system is attached to a single chain, then it is appropriate
to name the compounds as cycloalkylalkanes. For example,

SOLVED PROBLEM 4: Give the names for the following substituted alkanes:

Answer: (a) 1,1-dimethylethylcyclopentane or tert-butylcyclopentane; (b) 1-methyl-2-(2-


methylpropyl)cyclohexane or 1-isobutyl-2-methylcyclohexane; (c) butylcyclohexane

Nomenclature of Alkyl Halides or Haloalkanes

1. Alkanes bearing halogen substituents are named in the IUPAC substitutive system as
haloalkanes:

2. When the parent chain has both a halo and an alkyl substituent attached to it, number the
chain from the end nearer the first substituent, regardless of whether it is halo or alkyl. If two
substituents are at equal distance from the end of the chain, then number the chain from the
end nearer the substituent that has alphabetical precedence:

3. Common names for many simple haloalkanes are still widely used, however. In this common
nomenclature system, called functional class nomenclature, haloalkanes are named as alkyl
halides. (The following names are also accepted by IUPAC.)

B. Alkenes contain at least one carbon-carbon double bond (C═C), and this is indicated in
the family name and in names for specific compounds by an -ene ending.

Author: CRISTOBERT C. AYATON


School/Station: LIANGA NATIONAL COMPREHENSIVE HIGH SCHOOL – LIANGA I DISTRICT
Division: SURIGAO DEL SUR
email address: cristobert.ayaton@deped.gov.ph
12|Page
 Alkenes, also called olefins, have the general formula CnH2n where n=2, 3, ….
 Alkenes are classified as unsaturated hydrocarbons as opposed to alkanes which are
saturated hydrocarbons.
 Examples are ethene (C2H4) and propene (C3H6) – the two simplest alkenes.
 Alkenyl halide is a compound with a halogen atom bonded to an alkene carbon. In older
nomenclature such compounds were sometimes referred to as vinyl halides.

Nomenclature of Alkenes and Cycloalkenes

 The IUPAC rules for naming alkenes are similar in many respects to those for naming
alkanes:

1. Determine the parent name by selecting the longest chain that contains the double bond and
change the ending of the name of the alkane of identical length from -ane to -ene. Thus, if the
longest chain contains five carbon atoms, the parent name for the alkene is pentene; if it
contains six carbon atoms, the parent name is hexene, and so on.

2. Number the chain so as to include both carbon atoms of the double bond and begin
numbering at the end of the chain nearer the double bond. Designate the location the double
bond by using the number of the first atom of the double bond as a prefix. The locant for the
alkene suffix may precede the parent name or be placed immediately before the suffix. We will
show examples of both styles:

3. Indicate the locations of the substituent groups by the numbers of the carbon atoms to which
they are attached:

Many older names for alkenes are still in common use. Ethene is often called ethylene, propene
is often called propylene, and 2-methylpropene is often called isobutylene.

4. Number substituted cycloalkenes in the way that gives the carbon atoms of the double bond
the 1 and 2 positions and that also gives the substituent groups the lower numbers at the first
point of difference. With substituted cycloalkenes it is not necessary to specify the position of
the double bond since it will always begin with C1 and C2. The two examples shown here
illustrate the application of these rules:

Author: CRISTOBERT C. AYATON


School/Station: LIANGA NATIONAL COMPREHENSIVE HIGH SCHOOL – LIANGA I DISTRICT
Division: SURIGAO DEL SUR
email address: cristobert.ayaton@deped.gov.ph
13|Page

5. Name compounds containing a double bond and an alcohol group as alkenols (or
cycloalkenols) and give the alcohol carbon the lower number:

6. Two frequently encountered alkenyl groups are the vinyl group and the allyl group:

Using substitutive nomenclature, the vinyl and allyl groups are called ethenyl and prop-2-en-1-
yl, respectively. The following examples illustrate how these names are employed:

7. If two identical or substantial groups are on the same side of the double bond, the compound
can be designated cis; if they are on opposite sides it can be designated trans:

SOLVED PROBLEM 1: Give the IUPAC name for the following alkene:

Strategy and Answer: The longest chain contains 7 carbon atoms; thus, the parent name is
heptene. The double bond is located at C3 (between C3 and C4, numbered from left to right),
hence the IUPAC name is 3-heptene.

SOLVED PROBLEM 2: Give the IUPAC name for the following alkene:

Strategy and Answer: The longest chain contains 6 carbon atoms; thus, the parent name is
hexene. Substituents present are methyl, located at C2, and ethyl, located at C4. The double
bond is located at C1 (between C1 and C2, numbered from left to right), hence the IUPAC name
is 4-ethyl-2-methyl-1-hexene or 4-ethyl-2-methylhex-1-ene.

Author: CRISTOBERT C. AYATON


School/Station: LIANGA NATIONAL COMPREHENSIVE HIGH SCHOOL – LIANGA I DISTRICT
Division: SURIGAO DEL SUR
email address: cristobert.ayaton@deped.gov.ph
14|Page
SOLVED PROBLEM 3: Give the IUPAC name for the molecule shown.

Strategy and Answer: We number the ring as shown below starting with the hydroxyl group so
as to give the double bond the lower possible number. We include in the name the substituent
(an ethenyl group) and the double bond (-ene-), and the hydroxyl group (-ol) with numbers for
their respective positions. Hence the IUPAC name is 3-ethenyl-2-cyclopenten-1-ol.

C. Alkynes contain at least one carbon-carbon triple bond (C≡C), and this is indicated in the
family name and in names for specific compounds by an -yne ending.

 Alkynes have the general formula CnH2n-2 where n=2, 3, …


 Together with alkenes, alkynes are considered unsaturated hydrocarbons.
 Examples are ethyne (C2H2, commonly called acetylene), propyne (C3H4), and butyne (C4H6).

Nomenclature of Alkynes

 Alkynes are named in much similar way as alkenes.


1. Unbranched alkynes, for example, are named by replacing the -ane of the name of the
corresponding alkane with the ending -yne. The chain is numbered to give the carbon atoms of
the triple bond the lower possible numbers. The lower number of the two carbon atoms of the
triple bond is used to designate the location of the triple bond.

2. When double and triple bonds are present, the direction of numbering is chosen so as to give
the lowest overall set of locants. In the face of equivalent options, then preference is given
to assigning lowest numbers to the double bonds.

3. The locations of substituent groups of branched alkynes and substituted alkynes are also
indicated with numbers. An −OH group has priority over the triple bond when numbering the
chain of an alkynol:

SOLVED PROBLEM 1: Give the IUPAC name for the following alkyne:

Strategy and Answer: The longest chain contains 7 carbon atoms and numbered as shown
below. Methyl substituent is located at C5 and the triple bond is located at C1 (between C1 and
C2). Hence, the name of the alkyne is 5-methyl-1-hexyne or 5-methylhex-1-yne.

Author: CRISTOBERT C. AYATON


School/Station: LIANGA NATIONAL COMPREHENSIVE HIGH SCHOOL – LIANGA I DISTRICT
Division: SURIGAO DEL SUR
email address: cristobert.ayaton@deped.gov.ph
15|Page

SOLVED PROBLEM 2: Give the IUPAC name for the following alkyne:

Strategy and Answer: The longest chain contains 5 carbon atoms and numbered as shown
below. Two methyl substituents are both located at C4 and the triple bond is located at C1
(between C1 and C2). Hence, the name of the alkyne is 4,4-dimethyl-1-pentyne or 4,4-
dimethylpent-1-yne.

SOLVED PROBLEM 3: Give the IUPAC name for the following alkyne:

Strategy and Answer: The longest chain contains 5 carbon atoms and numbered as shown
below so as to give -OH group the lower number. Methyl and -OH substituents are both located
at C2 and the triple bond is located at C1 (between C1 and C2). Hence, the name of the alkyne
is 2-methyl-4-pentyn-2-ol or 2-methylpent-4-yn-2-ol.

D. Aromatic hydrocarbons contain a special type of ring, the most common example of which
is a benzene ring. There is no special ending in names for the general family of aromatic
compounds.

Phenyl and Benzyl Groups

 When a benzene ring is attached to some other group of atoms in a molecule, it is called a
phenyl group, and it is represented in several ways:

 The combination of a phenyl group and a methylene group (−CH2−) is called a benzyl group:

Author: CRISTOBERT C. AYATON


School/Station: LIANGA NATIONAL COMPREHENSIVE HIGH SCHOOL – LIANGA I DISTRICT
Division: SURIGAO DEL SUR
email address: cristobert.ayaton@deped.gov.ph
16|Page

 Aryl halide is a compound with a halogen atom bonded to an aromatic ring such as a benzene
ring. If the aromatic ring is benzene, the aryl halide is called phenyl halide.

 Functional groups are common and specific arrangements of atoms or bonds that impart
predictable reactivity and properties to a molecule.

 The C═C double bonds in alkenes and C≡C triple bonds in alkynes are just two of the many
functional groups in organic molecules. The C―C single bonds and C―H in alkanes are not
considered functional groups because these bonds are present in molecules of almost all
organic compounds and are, in general, much less reactive than common functional groups.

 Each kind of functional group often undergoes the same kinds of reactions in every molecule,
regardless of the size and complexity of the molecule. Thus, the chemistry of an organic
molecule is largely determined by the functional groups it contains.

 Classes of Organic Compounds and their Functional Groups

1. ALCOHOLS AND PHENOLS

 Alcohols and phenols have the functional group ―OH called either the hydroxyl group or the
alcohol group which is attached to a sp3-hybridized carbon atom.
 Alcohols have the general formula ROH.
 The simplest alcohol is methanol, CH3OH. Another example of an alcohol is CH3CH2OH called
ethanol or ethyl alcohol.

Specific Example
Functional Compound
Suffix Systematic Name
Group Type or Class Structural Formula Ball-and-stick Model
(Common Name)

 Two ways in viewing an alcohol structurally: (1) as hydroxyl derivative of alkanes and (2) as
alkyl derivatives of water. Ethyl alcohol, for example can be seen (1) as an ethane molecule
in which one hydrogen has been replaced by a hydroxyl group or (2) as a water molecule in
which one hydrogen has been replaced by an ethyl group.
 As with alkyl halides, alcohols are classified into three groups: primary (1°), secondary (2°),
or tertiary (3°) alcohols. This classification is based on the degree of substitution of the
carbon to which the hydroxyl group is directly attached.

A. 1° alcohol – If the carbon atom bearing the hydroxyl group has only one other carbon
attached to it, the carbon is said to be a primary carbon and the alcohol is a primary
alcohol.

Author: CRISTOBERT C. AYATON


School/Station: LIANGA NATIONAL COMPREHENSIVE HIGH SCHOOL – LIANGA I DISTRICT
Division: SURIGAO DEL SUR
email address: cristobert.ayaton@deped.gov.ph
17|Page

B. 2° alcohol – If the carbon atom bearing the hydroxyl group also has other two
carbon atoms attached to it, the carbon is said to be a secondary carbon and the
alcohol is a secondary alcohol.

C. 3° alcohol – If the carbon atom bearing the hydroxyl group has three other
carbons attached to it, this carbon is called a tertiary carbon and the alcohol is a
tertiary alcohol.

 When a hydroxyl group is bonded to a benzene ring, the combination of the ring and the
hydroxyl group is called a phenol. Phenols differ significantly from alcohols in terms of their
relative acidity (phenols being relatively acidic than alcohols) and thus they are considered a
distinct functional group.

 The general formula for phenols is ArOH.

Nomenclature of Alcohols

 The following procedure should be followed in giving alcohols IUPAC substitutive names:

1. Select the longest continuous carbon chain to which the hydroxyl is directly attached. Change
the name of the alkane corresponding to this chain by dropping the final -e and adding the
suffix -ol.

2. Number the longest continuous carbon chain so as to give the carbon atom bearing the
hydroxyl group the lower number. Indicate the position of the hydroxyl group by using this
number as a locant; indicate the positions of other substituents (as prefixes) by using the
numbers corresponding to their positions along the carbon chain as locants. The following
examples show how these rules are applied:

Author: CRISTOBERT C. AYATON


School/Station: LIANGA NATIONAL COMPREHENSIVE HIGH SCHOOL – LIANGA I DISTRICT
Division: SURIGAO DEL SUR
email address: cristobert.ayaton@deped.gov.ph
18|Page

SOLVED PROBLEM 1: Give the IUPAC name for the following alcohol:

Strategy and Answer: The longest chain contains 3 carbon atoms and numbered as shown
below. A chlorine substituent is located at C3 and the hydroxyl group is located at C1. Hence,
the name of the alcohol is 3-chloro-1-propanol or 3-chloropropan-1-ol.

SOLVED PROBLEM 2: Give the IUPAC name for the following alcohol:

Strategy and Answer: The longest chain contains 5 carbon atoms and numbered as shown
below. Two methyl substituents are both located at C4 and the hydroxyl group is located at C2.
Hence, the name of the alcohol is 4,4-dimethyl-2-pentanol or 4,4-dimethylpropan-2-ol.

SOLVED PROBLEM 3: Give the IUPAC name for the compound shown:

Strategy and Answer: We find that the longest carbon chain (in red at right) has five carbons
and it bears a hydroxyl group on the first carbon. So, we name this part of the molecule as a 1-
pentanol. There is a phenyl group on C1 and a methyl group on C3, so the full name is 3-
methyl-1-phenyl-1-pentanol or 3-methyl-1-phenylpentan-1-ol.

 Simple alcohols are often called by common functional class names that are also approved
by IUPAC. We have seen several examples already (Section 2.6). In addition to methyl alcohol,
ethyl alcohol, and isopropyl alcohol, there are several others, including the following:

 Alcohols containing two hydroxyl groups are commonly called glycols. In the IUPAC
substitutive system, they are named as diols:

Author: CRISTOBERT C. AYATON


School/Station: LIANGA NATIONAL COMPREHENSIVE HIGH SCHOOL – LIANGA I DISTRICT
Division: SURIGAO DEL SUR
email address: cristobert.ayaton@deped.gov.ph
19|Page

2. ETHERS

 Ethers have the general formula ROR’ where R’ is an alkyl or phenyl group that may be same
or different from R.

Specific Example
Functional Compound
Suffix Systematic Name
Group Type or Class Structural Formula Ball-and-stick Model
(Common Name)

 Ethers can be thought of as derivatives of water in which both hydrogen atoms have been
replaced by alkyl groups. The bond angle at the oxygen atom of an ether is only slightly larger
than that of water.
 Examples of ethers are CH3OCH3 called methoxymethane ( or dimethyl ether) and
CH3CH2OCH3 called methoxyethane ( or ethyl methyl ether).

Nomenclature of Ethers

 Simple ethers are frequently given common functional class names. One simply lists (in
alphabetical order) both groups that are attached to the oxygen atom and adds the word
ether:

 IUPAC substitutive names should be used for complicated ethers, however, and for
compounds with more than one ether linkages.

1. In this IUPAC style, ethers are named as alkoxyalkanes, alkoxyalkenes, and alkoxyarenes.
The −OR group is an alkoxy group.

Methoxyethane Ethoxyethane tert-Butoxybenzene

SOLVED PROBLEM 1: Give the IUPAC name for the following ether:

Strategy and Answer: The alkoxy group is methoxy group that is attached to C2 of the pentane
parent chain. Hence, the name of the ether is 2-methoxypentane.

SOLVED PROBLEM 2: Give the IUPAC name for the following ether:

Strategy and Answer: Two methoxy groups are attached to each carbon of the ethane parent
chain. Hence, the name of the ether is 1,2-dimethoxyethane.

Author: CRISTOBERT C. AYATON


School/Station: LIANGA NATIONAL COMPREHENSIVE HIGH SCHOOL – LIANGA I DISTRICT
Division: SURIGAO DEL SUR
email address: cristobert.ayaton@deped.gov.ph
20|Page

SOLVED PROBLEM 3: Give the IUPAC name for the following ether:

Strategy and Answer: Ethoxy group is located at C1 and a methyl group is located at C4 of a
benzene ring. Hence, the name of the ether is 1-ethoxy-4-methylbenzene.

3. AMINES

 Amines have the general formula RNH2, RNHR’, or RNR’R”.

Specific Example
Functional Compound
Suffix Systematic Name
Group Type or Class Structural Formula Ball-and-stick Model
(Common Name)

 Amines may be considered as organic derivatives of ammonia (NH3).

 The other two hydrogens in the amino (―NH2) group can be replaced by organic groups.
Amines are classified as primary (1°), secondary (2°), or tertiary (3°) amines. This
classification is based on the number of organic groups that are attached to the
nitrogen atom.

 Example of an amine is (CH3)2CHNH2 called 2-propanamine or isopropylamine. It is a primary


amine even though its ―NH2 group is attached to a 2° carbon atom because only one organic
group is attached to the nitrogen atom. Another example of an amine is (CH2)5NH called
piperidine.

Nomenclature of Amines

1. In systematic nomenclature they are named by adding the suffix -amine to the name of the
chain or ring system to which the NH2 group is attached with replacement of the final -e.

Methanamine Ethanamine

Author: CRISTOBERT C. AYATON


School/Station: LIANGA NATIONAL COMPREHENSIVE HIGH SCHOOL – LIANGA I DISTRICT
Division: SURIGAO DEL SUR
email address: cristobert.ayaton@deped.gov.ph
21|Page

2. Most secondary and tertiary amines are named in the same general way. In systematic
nomenclature we use the locant N to designate substituents attached to a nitrogen atom.

N-methylethanamine N,N-diethylethanamine

SOLVED PROBLEM 1: Give the IUPAC name for the following amine:

Strategy and Answer: The longest chain contains 3 carbon atoms. The methyl substituent is
located at C2 and the amino group is attached to C1. Hence, the name of the amine is 2-methyl-
1-propanamine.

SOLVED PROBLEM 2: Give the IUPAC name for the following amine:

Strategy and Answer: The parent chain contains 2 carbon atoms. One H in the -NH2 group is
replaced by an ethyl substituent. Hence, the name of the amine is N-ethylethanamine.

SOLVED PROBLEM 3: Give the IUPAC name for the following amine:

Strategy and Answer: The longest chain contains 3 carbon atoms. Two Hs in the -NH2 group
are each replaced by methyl and ethyl substituents. Hence, the name of the amine is N-ethyl-
N-methyl-1-propanamine.

ALDEHYDES AND KETONES

 Aldehydes and ketones both contain the same functional group called the carbonyl group –
a group in which the carbon atom has a double bond to an oxygen atom.

4. ALDEHYDES

 The carbonyl group of an aldehyde is bonded to one carbon atom and one hydrogen atom
(except for formaldehyde, which is the only aldehyde bearing two hydrogen atoms).
 Aldehydes have the general formula RCHO. For formaldehyde, R is replaced with H.
 Examples of aldehydes are HCHO called methanal or formaldehyde and CH3CHO called
ethanal or acetaldehyde.
Specific Example
Functional Compound
Suffix Systematic Name
Group Type or Class Structural Formula Ball-and-stick Model
(Common Name)

Author: CRISTOBERT C. AYATON


School/Station: LIANGA NATIONAL COMPREHENSIVE HIGH SCHOOL – LIANGA I DISTRICT
Division: SURIGAO DEL SUR
email address: cristobert.ayaton@deped.gov.ph
22|Page

Nomenclature of Aldehydes

 Aliphatic aldehydes are named in the IUPAC substitutive system by replacing the final -e of
the name of the corresponding alkane with -al.

1. Since the aldehyde group must be at an end of the carbon chain, there is no need to indicate
its position. When other substituents are present, the carbonyl group carbon is assigned
position 1.

2. Aldehydes in which the −CHO group is attached to a ring system are named by adding the
suffix carbaldehyde.

SOLVED PROBLEM 1: Give the IUPAC name for the following aldehyde:

Strategy and Answer: The parent chain contains 5 carbon atoms thus the parent name is
pentanal. A chloro substituent is attached to C5. Hence, the name of the aldehyde is 5-
chloropentanal.

SOLVED PROBLEM 2: Give the IUPAC name for the following aldehyde:

Strategy and Answer: The parent chain contains 2 carbon atoms thus the parent name is
ethanal. There is a phenyl substituent attached to C2 but we do not need to indicate 2- as a
locant because there is no other way that the phenyl group can the attached to an ethanal. Also,
we do not name the compound as carbaldehyde because the ring substituent is not directly
attached to the carbonyl carbon atom. Hence, the name of the aldehyde is phenylethanal.

SOLVED PROBLEM 3: Give the IUPAC name for the following aldehyde:

Strategy and Answer: The cyclohexane ring is directly attached to the carbonyl carbon atom,
hence the name of the aldehyde is cyclohexanecarbaldehyde.

5. KETONES

 The carbonyl group of a ketone is bonded to two carbon atoms.

Author: CRISTOBERT C. AYATON


School/Station: LIANGA NATIONAL COMPREHENSIVE HIGH SCHOOL – LIANGA I DISTRICT
Division: SURIGAO DEL SUR
email address: cristobert.ayaton@deped.gov.ph
23|Page
 Ketones have the general formula RCOR’ where R’ is an alkyl group that may be same or
different from R.
 Example of a ketone is CH3COCH3 called propanone or commonly known as acetone and is
considered the simplest ketone.

Compound Specific Example


Functional
Type or Suffix Systematic Name
Group Structural Formula Ball-and-stick Model
Class (Common Name)

Nomenclature of Ketones

 Aliphatic ketones are named by replacing the final -e of the name of the corresponding alkane
with -one.

1. The chain is numbered in the way that gives the carbonyl carbon atom the lower possible
number, and this number is used to designate its position.

SOLVED PROBLEM 1: Give the IUPAC name for the following ketone:

Strategy and Answer: The parent chain contains 5 carbon atoms numbered so as to give the
ketone functional group the lowest possible number, that is at C2. A double bond is located at
C4 (between C4 and C5). Hence, the name of the ketone is 4-penten-2-one or pent-4-en-2-one,
and not 1-penten-4-one.

SOLVED PROBLEM 2: Give the IUPAC name for the following ketone:

Strategy and Answer: The parent chain contains 2 carbon atoms and a phenyl substituent is
attached to C1, the carbonyl carbon atom. Hence, the name of the ketone is 1-phenylethanone.

CARBOXYLIC ACIDS, ESTERS, AND AMIDES

 Carboxylic acids, esters, and amides all contain a carbonyl group that is bonded to an oxygen
or nitrogen atom.

6. CARBOXYLIC ACIDS

 The functional group present in carboxylic acids is ―COOH called carboxyl group (carbonyl
+ hydroxyl) – a carbonyl group bonded to a hydroxyl group.

Author: CRISTOBERT C. AYATON


School/Station: LIANGA NATIONAL COMPREHENSIVE HIGH SCHOOL – LIANGA I DISTRICT
Division: SURIGAO DEL SUR
email address: cristobert.ayaton@deped.gov.ph
24|Page

 Carboxylic acids have the general formula R―COOH (or RCOOH).

Compound Specific Example


Functional
Type or Suffix Systematic Name
Group Structural Formula Ball-and-stick Model
Class (Common Name)

 Examples of carboxylic acids are methanoic acid (or formic acid), ethanoic acid (or acetic acid),
and benzoic acid.
 Formic acid is an irritating liquid produced by ants. (The sting of the ant is caused, in part,
by formic acid being injected under the skin. Formic is the Latin word for ant.) Acetic acid,
the substance responsible for the sour taste of vinegar, is produced when certain bacteria act
on the ethyl alcohol of wine and cause the ethyl alcohol to be oxidized by air.

Nomenclature of Carboxylic Acids

 Systematic or substitutive names for carboxylic acids are obtained by dropping the final -e of
the name of the alkane corresponding to the longest chain in the acid and by adding -oic acid.
The carboxyl carbon atom is assigned number 1.

Methanoic acid Ethanoic acid

Benzoic acid

SOLVED PROBLEM 1: Give the IUPAC name for the following carboxylic acid:

Strategy and Answer: The longest chain contains 6 carbon atoms thus the parent name is
hexanoic acid. A methyl substituent is located at C4 hence, the name is 4-methylhexanoic acid.

SOLVED PROBLEM 2: Give the IUPAC name for the following carboxylic acid:

Author: CRISTOBERT C. AYATON


School/Station: LIANGA NATIONAL COMPREHENSIVE HIGH SCHOOL – LIANGA I DISTRICT
Division: SURIGAO DEL SUR
email address: cristobert.ayaton@deped.gov.ph
25|Page
Strategy and Answer: The longest chain contains 7 carbon atoms with double bond at C3
hence, the name is 3-heptenoic acid or hept-3-enoic acid.

SOLVED PROBLEM 3: Give the IUPAC name for the following compound.

Strategy and Answer: This chain contains six carbons with one double bond, so the base name
is hexenoic acid. Then we give the position of the double bond at C2, and a chloro substituent
at C5. The name, therefore, is 5-chloro-2-hexenoic acid.

7. ESTERS

 Esters have the general formula RCOOR’ where the carbonyl group is bonded to an alkoxyl
group (―OR).

Compound Specific Example


Functional
Type or Suffix Systematic Name
Group Structural Formula Ball-and-stick Model
Class (Common Name)

 Example of an ester is ethyl ethanoate and pentyl butanoate (smells like apricots and pears).

Ethyl ethanoate Pentyl butanoate

 Our body makes esters from long-chain carboxylic acids called “fatty acids” by combining
them with glycerol.

Nomenclature of Esters

 The names of esters are derived from the names of the alcohol (with the ending -yl) and the
acid (with the ending -ate or -oate). The portion of the name derived from the alcohol comes
first:

SOLVED PROBLEM 1: Give the IUPAC name for the following ester:

Author: CRISTOBERT C. AYATON


School/Station: LIANGA NATIONAL COMPREHENSIVE HIGH SCHOOL – LIANGA I DISTRICT
Division: SURIGAO DEL SUR
email address: cristobert.ayaton@deped.gov.ph
26|Page

Strategy and Answer: The alcohol derivative is 3-methylbutyl or isopentyl, and the carboxylic
acid derivative contains 2 carbon atoms. Hence, the name of the ester is 3-methylbutyl
ethanoate or isopentyl ethanoate. Isopentyl ethanoate is used in synthetic banana flavor.

SOLVED PROBLEM 2: Give the IUPAC name for the following ester:

Strategy and Answer: The alcohol derivative is 3-methylbutyl or isopentyl, and the portion that
is a carboxylic acid derivative contains 5 carbon atoms in straight-chain. Hence, the name of
the ester is 3-methylbutyl pentanoate or isopentyl pentanoate. Isopentyl pentanoate is used in
synthetic apple flavor.

SOLVED PROBLEM 3: Give the IUPAC name for the following ester:

Answer: Methyl benzoate

8. AMIDES

 Amides have the general formula RCONH2, RCONHR’, or RCONR’R” where a carbonyl group
is bonded to a nitrogen atom bearing (1) two hydrogens; (2) hydrogen and alkyl group; or (3)
two alkyl groups.

Compound Specific Example


Functional
Type or Suffix Systematic Name
Group Structural Formula Ball-and-stick Model
Class (Common Name)

 Some examples of amide are the following:

Author: CRISTOBERT C. AYATON


School/Station: LIANGA NATIONAL COMPREHENSIVE HIGH SCHOOL – LIANGA I DISTRICT
Division: SURIGAO DEL SUR
email address: cristobert.ayaton@deped.gov.ph
27|Page

Nomenclature of Amides

1. Amides that have no substituent on nitrogen are named by dropping -ic acid from the
common name of the acid (or -oic acid from the substitutive name) and then adding -amide.

Ethanamide
2. Alkyl groups on the nitrogen atom of amides are named as substituents, and the named
substituent is prefaced by N- or N,N-.

N-methylethanamide N,N-dimethylethanamide

SOLVED PROBLEM 1: Give the IUPAC name for the following amide:

Strategy and Answer: The longest chain contains 5 carbon atoms, and a methyl substituent is
located at C2. Hs in -NH2 group are not replaced by any substituent. Hence, the name of the
amide is 2-methylpentanamide.

SOLVED PROBLEM 2: Give the IUPAC name for the following amide:

Strategy and Answer: The parent chain contains 2 carbon atoms. Phenyl and propyl
substituents replace the Hs in -NH2 group. Hence, the name of the amide is N-phenyl-N-
propylethanamide.

Exercises / Activities

Activity 1 – Multiple Choice Questions


What to do: Choose the letter of the correct answer.

1. Which of the following is not an organic compound?


A. C2H5OH C. CO2
B. CO(NH2)2 D. CH3OCH3

2. Which of the following does not exist as a hydrocarbon?


A. Pentane C. 2-Hexene
B. Methene D. 3-Nonyne

Author: CRISTOBERT C. AYATON


School/Station: LIANGA NATIONAL COMPREHENSIVE HIGH SCHOOL – LIANGA I DISTRICT
Division: SURIGAO DEL SUR
email address: cristobert.ayaton@deped.gov.ph
28|Page

3. What is the IUPAC name for the following branched alkane?

A. 4-Ethyl-2,6-diisopropylnonane
B. 6-Ethyl-4,8-diisopropylnonane
C. 6-Ethyl-4-isopropyl-8,9-dimethyldecane
D. 5-Ethyl-7-isopropyl-2,3-dimethyldecane

4. What is the IUPAC name for the compound whose bond-line structural formula is given
below?

A. 5-Bromo-3-chloro-6-ethylheptane
B. 3-Bromo-5-chloro-2-ethylheptane
C. 4-Bromo-6-chloro-3-methyloctane
D. 1-Bromo-3-chloro-1-(1-Methylpropyl)pentane

5. What is the IUPAC name for the compound whose bond-line structural formula is given
below?

A. 1-Chloro-2,4-dimethylcyclohexane
B. 4-Chloro-1,5-dimethylcyclohexane
C. 6-Chloro-1,3-dimethylcyclohexane
D. 1-Chloro-4,6-dimethylcyclohexane

6. What is the IUPAC name for the following branched alkene?

A. 5-Isopropyl-2-hexene
B. 5,6-Dimethyl-2-heptene
C. 2,3-Dimethyl-5-heptene
D. 5,6,6-Trimethyl-2-hexene

7. What is the IUPAC name for the following branched alkyne?


CH3C(CH3)2CH2C≡CCH3

A. 2-Octyne
B. 6-Octyne
C. 5,5-Dimethyl-2-hexyne
D. 2,2-Dimethyl-4-hexyne

8. What is the IUPAC name for the following alcohol?

A. 2,4-Dimethyl-5-pentanol
B. 2,4-Dimethyl-1-pentanol
C. 3-Isopropyl-2-methyl-1-propanol
D. 1-Isopropyl-2-methyl-3-propanol

9. What is the IUPAC name for the following ether?

A. Propoxyethane

Author: CRISTOBERT C. AYATON


School/Station: LIANGA NATIONAL COMPREHENSIVE HIGH SCHOOL – LIANGA I DISTRICT
Division: SURIGAO DEL SUR
email address: cristobert.ayaton@deped.gov.ph
29|Page
B. 2-Ethoxypropane
C. Isopropoxyethane
D. 1-methylethoxyethane
10. What is the IUPAC name for the following amine?

A. N,N-Diethyl-2-ethyl-1-propanamine
B. N,N-Diethyl-2-methyl-1-butanamine
C. N,N-Dimethyl-2-ethyl-1-butanamine
D. N,N-Dimethyl-2-ethyl-1-propanamine

11. What is the IUPAC name for the following aldehyde?

A. Butanal
B. Propanal
C. 3-Methylbutanal
D. 2-Methylpropanal

12. What is the IUPAC name for the following ketone?

A. 6-Methyl-3-heptanone
B. 2-Methyl-5-heptanone
C. 5-isopropyl-3-hexanone
D. 6,6-Dimethyl-3-hexanone

13. What is the IUPAC name for the following carboxylic acid?

A. 1-Phenylhexanoic acid
B. 5-Phenylhexanoic acid
C. 1-Phenylpentanoic acid
D. 5-Phenylpentanoic acid

14. What is the IUPAC name for the following ester?

A. Ethyl benzoate
B. Propyl benzoate
C. Phenyl ethanoate
D. Phenyl propanoate

15. What is the IUPAC name for the following amide?

A. N-Cyclopropyl-5-methylhexanamide
B. N-Cyclopropyl-4-methylpentanamide
C. N-Cyclopropyl-4-isopropylbutanamide
D. N-Cyclopropyl-3-isopropylpropanamide

Activity 2 – What Am I?
What to do: Identify each of the following compounds as an alkane, alkene, alkyne, alcohol,
aldehyde, amine, and so forth.

Author: CRISTOBERT C. AYATON


School/Station: LIANGA NATIONAL COMPREHENSIVE HIGH SCHOOL – LIANGA I DISTRICT
Division: SURIGAO DEL SUR
email address: cristobert.ayaton@deped.gov.ph
30|Page

Activity 3 – A Test You Can Taste


What to do: Identify all the functional groups present in Capsaicin, a compound found in chili
peppers that gives them their spicy kick.

***Note that the class of compound can be used to name the functional group present in a
compound. For example: for the class aldehydes and ketones, the functional group is carbonyl,
but when asked to identify functional groups present in a compound and it happens to have a
carbonyl, it is accepted to say aldehyde or ketone, instead of carbonyl, for specificity.

Capsaicin

Reflection: Choose one organic compound that can be found in your household and provide the
following information:
a. Chemical Name and Chemical Formula of the organic compound
b. Structure of the organic compound and identify the functional group/s present
c. Use of the organic compound in everyday life
d. Effects of the organic compound to humans and other living things
2 (NEEDS 4 (EXCEEDS
3 (MEETS
CRITERIA 1 (NOT VISIBLE) IMPROVEMENT) EXPECTATIONS)
EXPECTATIONS)
Few of the required
elements are clearly Most of the required All of the required
Missing most or all
REQUIRED visible, organized and elements are clearly elements are clearly
of the required
ELEMENTS visible, organized visible, organized
elements. well placed. May be
and well placed. and well placed.
missing elements.
The project needs The project has an
The project has a
significant The project needs excellent design and
VISUAL CLARITY nice design and
improvement in improvement in design, layout. It is neat and
AND APPEAL layout. It is neat and
design, layout and layout or neatness. easy to understand
easy to read.
neatness. the content.
The student’s Few of the drawings Most drawings and Drawings and
DRAWINGS/ and graphics are clear
drawings are not graphics are clear graphics are clear
GRAPHICS and relevant.
clear or relevant. and relevant. and relevant.
Almost no
Many grammatical, A few grammatical, No grammatical,
grammatical, spelling
MECHANICS spelling or spelling or punctuation spelling or
or punctuation
punctuation errors. errors. punctuation errors.
errors.
e. Safety precautions in using the organic compound

Rubric:

References for learners:

Brown, T. L., LeMay, H. E., Bursten, B. E., Murphy, C. J., & Woodward, P. M.
(2012). Chemistry: The Central Science. (13th ed.) Pearson Prentice Hall.

Patalinghug, Wyona C., Vic Marie I. Camacho, Fortunato B. Sevilla III, and Maria Cristina D.
Singson. 2016. Teaching Guide for Senior High School General Chemistry 1. Quezon City:
Commission on Higher Education.

Author: CRISTOBERT C. AYATON


School/Station: LIANGA NATIONAL COMPREHENSIVE HIGH SCHOOL – LIANGA I DISTRICT
Division: SURIGAO DEL SUR
email address: cristobert.ayaton@deped.gov.ph
31|Page

Solomons, G T.W., Fhryle, C. B., Synder, S. A. 2016. Organic Chemistry. (12th ed). Wiley.

Answer Key

Activity 1

1. C 6. B 11. D
2. B 7. C 12. A
3. D 8. B 13. D
4. C 9. B 14. D
5. A 10. B 15. B

Activity 2

a. Ketone d. Aldehyde
b. Alkyne e. Alcohol (2°)
c. Alcohol (2°) f. Alkene

Activity 3

Author: CRISTOBERT C. AYATON


School/Station: LIANGA NATIONAL COMPREHENSIVE HIGH SCHOOL – LIANGA I DISTRICT
Division: SURIGAO DEL SUR
email address: cristobert.ayaton@deped.gov.ph
1|Page
WEEKLY LEARNING ACTIVITY SHEETS
General Chemistry 1 Grade 11, Quarter 2, Week 6

Name: ___________________________________________ Section: ____________________

ORGANIC COMPOUNDS

Lesson 1: Structural Isomers

Learning Competency:

Describe structural isomerism; give examples (STEM_GC11OCIIg-j-89)

Specific Objective:

1. Define structural isomerism;


2. Recognize molecules that are structural isomers of each other given the structural formulas

Time allotment: 2 Hours

Key Concepts

 Isomers are different compounds that have the same molecular formula. Isomers are
categorized into structural (or constitutional) isomers and stereoisomers. In this lesson, we will
focus only on structural isomers.
 Structural isomers have the same molecular formula but have different connectivity, meaning
that their atoms are connected in different order. The terms structural isomers and constitutional
isomers can be used interchangeably.

SOLVED PROBLEM 1: There are two structural isomers of C4H10. What are these?

Strategy and Answer: (a) C4H10 is an of an alkane with n=4. A straight-chain alkane with 4 carbon
atoms is butane with the condensed structural formula CH3CH2CH2CH3. Butane has the following
dash structural formula and bond-line structural formula:

Dash formula Bond-line formula

Butane (C4H10)

(b) The other structural isomer of the compound having molecular formula C 4H10 is 2-
methylpropane, a branched alkane. The backbone chain is propane, and the substituent is a
methyl group attached to C2. 2-Methylpropane has the following dash structural formula and
bond-line structural formula:

Dash formula Bond-line formula

2-Methylpropane (C4H10)

Author: CRISTOBERT C. AYATON


School/Station: LIANGA NATIONAL COMPREHENSIVE HIGH SCHOOL – LIANGA I DISTRICT
Division: SURIGAO DEL SUR
email address: cristobert.ayaton@deped.gov.ph
2|Page
SOLVED PROBLEM 2: What are the two structural isomers of the compound having a molecular
formula C3H7Cl?

Strategy and Answer: (a) If we replace H in place of Cl, we could get C3H8 which is an alkane with
n=3. Therefore, C3H7Cl is an alkyl halide where one H atom is replaced by a Cl atom. A Cl atom
attached to any of the end carbon of propane is a 1° alkyl halide called 1-chloropropane. 1-
Chloropropane has the following structural formula and bond-line structural formula:

Dash formula Bond-line formula

1-Chloropropane (C3H7Cl)

(b) The other structural isomer of the compound having a molecular formula C3H7Cl is a 2° alkyl
halide called 2-chloropropane. 2-Chloropropane has the following structural formula and bond-
line structural formula:

Dash formula Bond-line formula

2-Chloropropane (C3H7Cl)

SOLVED PROBLEM 3: Give the seven structural isomers of the compound having a molecular
formula C4H10O.

Strategy and Answer:


Alcohols, ethers, aldehydes, and ketones are the classes of organic compounds that contain atoms
of carbon and hydrogen, and at least one atom of oxygen. For C4H10O, only alcohols and ethers
form structural isomers. The seven structural isomers of C4H10O are given below:

Alcohols:

Dash formula Bond-line formula Dash formula Bond-line formula

1-Butanol (C4H10O) 2-Butanol (C4H10O)

Dash formula Bond-line formula Dash formula Bond-line formula

2-Methyl-1-propanol (C4H10O) 2-Methyl-2-propanol (C4H10O)

Author: CRISTOBERT C. AYATON


School/Station: LIANGA NATIONAL COMPREHENSIVE HIGH SCHOOL – LIANGA I DISTRICT
Division: SURIGAO DEL SUR
email address: cristobert.ayaton@deped.gov.ph
3|Page
Ethers:

Dash formula Bond-line formula Dash formula Bond-line formula

1-Methoxypropane (C4H10O) 2-Methoxypropane (C4H10O)

Dash formula Bond-line formula

Ethoxyethane (C4H10O)

SOLVED PROBLEM 4: Propene, CH3CH═CH2, is an alkene. Write the structure of a constitutional


isomer of propene that is not an alkene. That means, it has no double bond.

Strategy and Answer:


An alkene with one double bond having n carbon atoms will have the same molecular formula as
a cycloalkane with the same number of carbons. For alkenes and cycloalkanes, the general formula
is CnH2n. Thus, cyclopropane is a constitutional isomer of propene.

Dash formula Bond-line formula Dash formula Bond-line formula

Cyclopropane (C3H6) Propene (C3H6)

SOLVED PROBLEM 5: Propose structures of two constitutional isomers of cyclopentene (C3H8)


that do not contain a ring.

Dash formula Bond-line formula

Cyclopentene (C5H8)

Strategy and Answer: A cycloalkene with n carbon atoms will have the same molecular formula
as an alkyne with the same number of carbon atoms. For alkynes and cycloalkenes, the general
formula is CnH2n-2 Thus, 1-pentyne and 2-pentyne are constitutional isomers of cyclopentene.

Dash formula Bond-line formula Dash formula Bond-line formula

1-pentyne (C5H8) 2-pentyne (C5H8)

Author: CRISTOBERT C. AYATON


School/Station: LIANGA NATIONAL COMPREHENSIVE HIGH SCHOOL – LIANGA I DISTRICT
Division: SURIGAO DEL SUR
email address: cristobert.ayaton@deped.gov.ph
4|Page
 WARNING: Be careful not to draw “false” isomers which are just twisted versions of the original
molecule. For example, the structure below is just the straight-chain version of butane (as in
SOLVED PROBLEM 1.1) rotated about the central carbon-carbon bond.

SOLVED PROBLEM 6: What is the relationship between these two compounds?

Strategy and Answer:


They might look different molecules, but they represent the same molecule. There is a 180°-
rotation around the bond between C3 and C4 in the heptane backbone chain.

Lesson 2: Simple Reactions of Organic Compounds

Learning Competency:

Describe some simple reactions of organic compounds: combustion of organic fuels, addition,
condensation, and saponification of fats (STEM_GC11OCIIg-j-90)

Specific Objective:

1. Describe and predict the major product of some simple reactions of organic compounds

Time Allotment: 2 Hours

Key Concepts

COMBUSTION OF ORGANIC FUELS

 Most alkanes are relatively unreactive primarily because of the strength and lack of polarity of
the C―C and C―H bonds present. At room temperature, for example, alkanes do not react with
acids, bases, and strong oxidizing agents. This low reactivity accounts for the fact that alkanes
were originally called paraffins (Latin: parum affinis, meaning “little affinity”).
 Alkanes are not completely inert, however. One of their most commercially important reactions
is combustion in air, which is the basis of their use as fuels.
 Generally, many hydrocarbons are used as fuels because their combustion releases very large
amounts of heat energy.
 Combustion is the reaction with oxygen to produce heat energy, carbon dioxide and water. This
reaction occurs, for example, in cylinders of automobiles, in furnaces, and, more gently, with
paraffin candles.
 Complete combustion pertains to the complete conversion of carbon to carbon dioxide. Mostly,
combustion reactions are described by complete combustion especially in theoretical
calculations, i.e. calculations in theoretical yield of carbon dioxide; but in reality, complete
combustion is difficult to achieve.
 Incomplete combustion is indicated by the presence of solid elemental carbon and carbon
monoxide in the product, which means that not all the carbon is converted to carbon dioxide.
This happens due to insufficient supply of oxygen in the process. Incomplete combustion

Author: CRISTOBERT C. AYATON


School/Station: LIANGA NATIONAL COMPREHENSIVE HIGH SCHOOL – LIANGA I DISTRICT
Division: SURIGAO DEL SUR
email address: cristobert.ayaton@deped.gov.ph
5|Page
calculations are mostly of industrial applications using material balance such as in the fields of
chemical engineering. In the following examples, however, combustion reactions are assumed
complete.

EXAMPLE 1: Complete combustion of ethane (C2H6) proceeds according to this reaction:

𝟐𝑪𝟐 𝑯𝟔 (𝒈) + 𝟕𝑶𝟐 (𝒈) → 𝟒𝑪𝑶𝟐 (𝒈) + 𝟔𝑯𝟐 𝑶(𝒍)

EXAMPLE 2: Liquefied Petroleum Gas (LPG) is composed of propane, butane, a little methane,
and some other hydrocarbons. Combustion of propane is represented by the reaction:

𝑪𝟑 𝑯𝟖 (𝒈) + 𝟓𝑶𝟐 (𝒈) → 𝟑𝑪𝑶𝟐 (𝒈) + 𝟒𝑯𝟐 𝑶(𝒍)

EXAMPLE 3: For an internal combustion engine to move a vehicle, it must convert the energy
stored in the fuel into mechanical energy to drive the wheels. Gasoline, one of the most
commonly used fuels, contain large amount of octane. Combustion of octane proceeds as
follows:
𝟐 𝑪𝟖 𝑯𝟏𝟖 (𝒍) + 𝟐𝟓𝑶𝟐 (𝒈) → 𝟏𝟔𝑪𝑶𝟐 (𝒈) + 𝟏𝟖𝑯𝟐 𝑶(𝒍)

EXAMPLE 4: Ethyne, C2H2, is used in oxyacetylene torches in welding metals. Combustion of


ethyne is represented by the reaction:

𝟐𝑪𝟐 𝑯𝟐 (𝒈) + 𝟓𝑶𝟐 (𝒈) → 𝟒𝑪𝑶𝟐 (𝒈) + 𝟐𝑯𝟐 𝑶(𝒍)

EXAMPLE 5: Ethanol, C2H5OH, can be used as a fuel source in an alcohol lamp. Combustion
of ethanol can be written as:

𝑪𝟐 𝑯𝟓 𝑶𝑯(𝒍) + 𝟑𝑶𝟐 (𝒈) → 𝟐𝑪𝑶𝟐 (𝒈) + 𝟑𝑯𝟐 𝑶(𝒍)

ADDITION REACTIONS OF ALKENES

 Addition reaction forms new bonds between new groups and each end of an alkene (as well as
alkyne) forming a single product.
 Generally, this type of reaction can be depicted using E for an electrophilic portion of a reagent
and Nu for a nucleophilic portion, as follows.

 Electrophiles seek electrons. They are attracted to sites of negative charge and have the
property of being electrophilic.
 Nucleophiles are electron donors. They are attracted to sites of positive charge and have the
property of being nucleophilic.
 Mechanism of addition reaction: (1) In the protonation (addition of proton, or H +) of an alkene,
the electrophile is the proton (H+) donated by the HX group and the nucleophile is the alkene.
The result is a carbocation (carbon cation) and a halide ion.

Electrophile Nucleophile Carbocation Halide ion

 (2) The new electrophile is the carbocation formed from the above reaction, and the new
nucleophile is the halide ion. In this step, the nucleophilic halide ion donates an electron pair
to the carbocation, completing the formation of the product which is haloalkane.

Author: CRISTOBERT C. AYATON


School/Station: LIANGA NATIONAL COMPREHENSIVE HIGH SCHOOL – LIANGA I DISTRICT
Division: SURIGAO DEL SUR
email address: cristobert.ayaton@deped.gov.ph
6|Page

Electrophile Nucleophile Haloalkane

 To promote the reaction, the reaction temperature must be raised 500°C, and a catalyst (such
as Pt, Pd, or Ni) is used to assist in rupturing the H―H bond. We write such conditions over the
reaction arrow to indicate they must be present in order for the reaction to occur. The most
widely used catalysts are finely divided metals on which H2 is adsorbed.

A. Electrophilic Addition of Hydrogen Halides to Alkenes: Mechanism and Markovnikov’s


Rule
 Hydrogen halides (HI, HBr, HCl, and HF) add to the double bond of alkenes producing an
alkyl halide.

 One way to state Markovnikov’s [pronounced as markofnikof’s] rule is to say that in the
addition of HX to an unsymmetrical alkene, the hydrogen atom adds to the carbon
atom of the double bond that already has the greater number of hydrogen atoms.

 The addition of HX to an unsymmetrical alkene could conceivably occur in two ways. However,
one product usually predominates. An unsymmetrical alkene has different groups attached
to each carbon atom in the carbon-carbon double bond of an alkene.

 Reactions like the Markovnikov additions of hydrogen halides to alkenes are described as
being regioselective. Regio comes from the Latin word regionem meaning direction. A
regioselective reaction is one that can potentially yield two or more constitutional isomers
but that actually produces only one (or a predominance of one).

SOLVED PROBLEM 1: What is the main product in the addition of HBr to propene?

Strategy and Answer: The addition of HBr to propene produces 1-bromopropane or 2-


bromopropane. Following Markovnikov’s rule, the hydrogen atom will attach to C1 having the
greater number of hydrogen atoms and the bromine atom will attach to C2. Thus, the main
product is 2-bromopropane. 1-bromopropane will be produced in little amount.

+ H―Br → ( )
Propene Hydrogen bromide 2-Bromopropane 1-Bromopropane
(main product) (little amount)

SOLVED PROBLEM 2: What is the main product when 2-methylpropene reacts with HBr?

Author: CRISTOBERT C. AYATON


School/Station: LIANGA NATIONAL COMPREHENSIVE HIGH SCHOOL – LIANGA I DISTRICT
Division: SURIGAO DEL SUR
email address: cristobert.ayaton@deped.gov.ph
7|Page
Strategy and Answer: The main product is 2-bromo-2-methylpropane, not 1-bromo-2-
methylpropane. Following Markovnikov’s rule, the hydrogen atom will attach to C1 and the
bromine atom will attach to C2.

+ H―Br → ( )
2-Methylpropene Hydrogen bromide 2-Bromo-2-methylpropane 1-Bromo-2methylpropane
(main product) (little amount)

SOLVED PROBLEM 3: Determine the product when ethene reacts with HCl.

Strategy and Answer: Ethene is a symmetrical alkene. Markovnikov’s rule cannot be applied
and there is only one product which is chloroethane.

+ H―Cl 
Ethene Hydrogen chloride Chloroethane

B. Hydrogenation of Alkenes

 Hydrogenation is simply an addition of hydrogen to reduce an alkene to an alkane. One molar


equivalent of hydrogen is required for such reaction.
 The general reaction is one in which the atoms of the hydrogen molecule add to each atom of
the carbon–carbon double bond of the alkene.

 The food industry makes use of catalytic hydrogenation to convert liquid vegetable oils to
semisolid fats in making margarine and solid cooking fats. Examine the labels of many
prepared foods and you will find that they contain “partially hydrogenated vegetable oils.”
There are several reasons why foods contain these oils, but one is that partially hydrogenated
vegetable oils have a longer shelf life.

SOLVED PROBLEM 1: What is the product of hydrogenation of ethene, C2H4?

Strategy and Answer: The product of hydrogenation of ethene is the alkane ethane, C2H6.

+ H―H 

Ethene Hydrogen Ethane

SOLVED PROBLEM 2: What alkane is formed when 2-methyl-1-propene (C4H8) undergoes


hydrogenation reaction?

Strategy and Answer: The alkane formed in the hydrogenation of 2-methyl-1-propene is 2-


methylpropane.

Author: CRISTOBERT C. AYATON


School/Station: LIANGA NATIONAL COMPREHENSIVE HIGH SCHOOL – LIANGA I DISTRICT
Division: SURIGAO DEL SUR
email address: cristobert.ayaton@deped.gov.ph
8|Page

+ H―H 

2-Methyl-1-propene Hydrogen 2-Methylpropane

SOLVED PROBLEM 3: What is the product of hydrogenation of cyclohexene (C6H10)?

Strategy and Answer: When cyclohexene undergoes hydrogenation reaction, the product is
cyclohexane.

+ H―H 

Cyclohexene Hydrogen Cyclohexane

ADDITION REACTIONS OF ALKYNES

 Addition reaction of alkynes proceeds with the same conditions and catalysts as the addition
reaction of alkenes.

A. Addition of Hydrogen Halides to Alkynes

 Alkynes react with one molar equivalent of hydrogen chloride or hydrogen bromide to form
haloalkenes, and with two molar equivalents to form geminal dihalides.
 Geminal dihalide has two the same halogen attached to a same carbon atom.
 Both additions are regioselective and follow Markovnikov’s rule:

SOLVED PROBLEM 1: What is the product when 1-hexyne undergoes addition reaction with
(a) one molar equivalent of hydrogen bromide; (b) two molar equivalents of hydrogen bromide.

Strategy and Answer: The carbon-carbon triple bond is located between C1 and C2. C1
contains one hydrogen atom while no hydrogen atom is attached to C2. By Markovnikov’s rule,
the hydrogen atom of hydrogen bromide will attach to C1 and the bromine atom will attach to
C2. (a) 1-Hexyne reacts with one molar equivalent of hydrogen bromide to yield 2-bromo-1-
hexene. (b) Addition of two molar equivalents of hydrogen bromide to 1-hexyne further reduces
2-bromo-1-hexene to 2,2-dibromohexane.

(a)

+ H―Br 

1-Hexyne Hydrogen bromide 2-Bromo-1-hexene

Author: CRISTOBERT C. AYATON


School/Station: LIANGA NATIONAL COMPREHENSIVE HIGH SCHOOL – LIANGA I DISTRICT
Division: SURIGAO DEL SUR
email address: cristobert.ayaton@deped.gov.ph
9|Page
(b)

+ 2 H―Br 

1-Hexyne Hydrogen bromide 2,2-Diromohexane

B. Hydrogenation of Alkynes

 Hydrogenation is simply an addition of one molar equivalent of hydrogen to reduce an alkyne


to alkene (or two molar equivalents of hydrogen to reduce an alkyne to alkane).
 The general reaction is one in which the atoms of the hydrogen molecule add to each atom of
the carbon–carbon triple bond of the alkyne.

SOLVED PROBLEM 1: What is the product of hydrogenation of ethyne, C2H2, with (a) one molar
equivalent of hydrogen; (b) two molar equivalents of hydrogen?

Strategy and Answer: Hydrogenation of ethyne with (a) one molar equivalent of hydrogen yields
ethene and with (b) two molar equivalents yield ethane.

𝐻2 𝐻2
→ →
Ethyne Ethene Ethane

CONDENSATION REACTION: ESTERIFICATION

 A condensation reaction is a reaction in which two molecules are joined and a small molecule,
often water, is usually removed during condensation reaction.
 Esterification is the formation of an ester by reaction of a carboxylic acid, acid chloride, or
carboxylic acid anhydride with an alcohol or phenol.

Acid-Catalyzed Esterification

 Esters can be formed by an acid-catalyzed reaction of a carboxylic acid with an alcohol or


phenol.

Carboxylic acid Alcohol Ester Water

 HA is an acid used as catalyst with the application of heat and the double arrow indicates
that the reaction is reversible.

SOLVED PROBLEM 1: What ester is formed in the acid-catalyzed esterification reaction of


ethanoic acid with ethanol?

Author: CRISTOBERT C. AYATON


School/Station: LIANGA NATIONAL COMPREHENSIVE HIGH SCHOOL – LIANGA I DISTRICT
Division: SURIGAO DEL SUR
email address: cristobert.ayaton@deped.gov.ph
10|Page
Strategy and Answer: Ethanoic acid reacts with ethanol, in the presence of an acid, to produce
an ester ethyl ethanoate and water.

Ethanoic acid Ethanol Ethyl ethanoate Water

SOLVED PROBLEM 2: What is the product in the acid-catalyzed esterification reaction of


benzoic acid and methanol?

Strategy and Answer: When benzoic acid reacts with methanol, the product is methyl benzoate
and water.

Benzoic acid Methanol Methyl benzoate Water

SAPONIFICATION: A BASE-PROMOTED HYDROLYSIS OF ESTERS

 Saponification is the reaction of esters with aqueous sodium hydroxide (NaOH) solution to
produce sodium salt of the carboxylic acid and an alcohol. It is a common reaction of esters.
 Sapo is a Latin word for soap. This reaction originates from soapmaking and was originally
produced by the hydrolysis of fats.

Ester Sodium hydroxide Sodium carboxylate Alcohol

SOLVED PROBLEM 1: What are the products of the base-promoted hydrolysis of ethyl
butanoate?

Strategy and Answer: Ethyl butanoate reacts with aqueous sodium hydroxide to produce
sodium butanoate and ethanol.

+ NaOH  +

Saponification of Triacylglycerol

 Another way to define saponification is that, saponification is the alkaline (basic condition)
hydrolysis of triacylglycerols (or triglycerides), leading to glycerol and a mixture of salts of
long-chain carboxylic acids.

Author: CRISTOBERT C. AYATON


School/Station: LIANGA NATIONAL COMPREHENSIVE HIGH SCHOOL – LIANGA I DISTRICT
Division: SURIGAO DEL SUR
email address: cristobert.ayaton@deped.gov.ph
11|Page

Triacylglycerol Glycerol Sodium carboxylates


“soap”

SOLVED PROBLEM 2: What are the products in the saponification of a triacylglycerol consisting
palmitic acid, CH3―(CH2)14―COOH as the fatty acid component?

Strategy and Answer: Saponification of triacylglycerols with 3 molar equivalents of sodium


hydroxide produces three molar equivalents of sodium salt of long-chain carboxylic acid. Along
with the backbone glycerol as the product, the reaction produces three molar equivalents of
sodium palmitate, CH3―(CH2)14―CO― Na+.

+ 3 NaOH  +

Glyceryl tripalmitate Sodium hydroxide Glycerol Sodium palmitate (Soap)

Lesson 1: Exercises / Activities

Activity 1 – Multiple Choice Questions


What to do: Choose the letter of the correct answer. Consider each pair of structural formulas
that follow and state whether the two formulas represent:
A. the same compound
B. different compounds that are constitutional isomers of each other
C. different compounds that are NOT isomeric

1. 5.

2. 6.

3. 7.

4. 8.

Author: CRISTOBERT C. AYATON


School/Station: LIANGA NATIONAL COMPREHENSIVE HIGH SCHOOL – LIANGA I DISTRICT
Division: SURIGAO DEL SUR
email address: cristobert.ayaton@deped.gov.ph
12|Page

9. 12.

10. 13.

11. 14.

15.

Activity 2
What to do: Answer as directed.

1. Write the balanced chemical equation for the combustion reaction of butane, C4H10.

2. What is the main product in the addition of HBr to 3-methyl-1-pentene? Write the chemical
equation using structural formulas of the reactants and the product.

3. What is the product of addition of two molar equivalents of HCl to 3-methyl-1-pentyne? Write
the chemical equation using structural formulas of the reactants and the product.

4. What is the product of hydrogenation of 2,3-dimethyl-1-heptyne with (a) one molar equivalent
of hydrogen, (b) two molar equivalents of hydrogen? Write the chemical equation using
structural formulas of the reactants and the product.

Reflection

1. Why do we need to study organic chemistry? Why do we need to study reactions of organic
compounds?
2. Almost everyday in human activity involves combustion of fuels which is one of the many causes
of anthropogenic (man-made) emissions of pollutants in our environment. As a student of
chemistry, what will you do to mitigate the effects of global warming and climate change due to
increasing carbon footprint in the atmosphere?

Rubric:
Meets Approaching Below
Above Expectations Expectations Expectations
Expectations (4) (3) (2) (1)
The reflection attempts
The reflection explains the The reflection
to demonstrate The reflection does
student’s own thinking explains the
thinking about learning not address the
and learning processes, asstudent’s thinking
Reflective but is vague and/or student’s thinking
well as implications for about his/her own
Thinking unclear about the and/or learning.
future learning. learning processes.
personal learning
process.

Author: CRISTOBERT C. AYATON


School/Station: LIANGA NATIONAL COMPREHENSIVE HIGH SCHOOL – LIANGA I DISTRICT
Division: SURIGAO DEL SUR
email address: cristobert.ayaton@deped.gov.ph
13|Page

The reflection is an in-


The reflection is
depth analysis of the The reflection attempts
an analysis of
learning experience, the to analyze the learning The reflection does
the learning
value of the derived experience but the not move beyond a
experience and
Analysis learning to self or others, value of the learning to description of the
the value of the
and the enhancement of the student or others is learning experience.
derived learning
the student’s appreciation vague and/or unclear.
to self or others.
for the discipline.

The reflection
The reflection attempts
articulates
to articulate
connections
The reflection articulates connections between
between this
multiple connections this learning The reflection does
learning
between this learning experience and content not articulate any
experience and
Making experience and content from other courses, connection to other
content from
Connections from other courses, past past learning learning or
other courses,
learning, life experiences experiences, or experiences.
past learning
and/or future goals. personal goals, but the
experiences,
connection is vague
and/or future
and/or unclear.
goals.

References for learners:


Brown, T. L., LeMay, H. E., Bursten, B. E., Murphy, C. J., & Woodward, P. M.
(2012). Chemistry: The Central Science. (13th ed.) Pearson Prentice Hall.

Patalinghug, Wyona C., Vic Marie I. Camacho, Fortunato B. Sevilla III, and Maria Cristina
D. Singson. 2016. Teaching Guide for Senior High School General Chemistry 1. Quezon City:
Commission on Higher Education.

Solomons, G T.W., Fhryle, C. B., Synder, S. A. 2016. Organic Chemistry. (12th ed). Wiley.

Answer Key

Activity 1
1. C 6. B 11. B
2. A 7. C 12. A
3. A 8. A 13. A
4. A 9. C 14. C
5. A 10. A 15. A

Activity 2

1. 𝟐𝑪𝟒 𝑯𝟏𝟎 + 𝟏𝟑𝑶𝟐 → 𝟖𝑪𝑶𝟐 + 𝟏𝟎𝑯𝟐 𝑶


2. 2-bromo-3-methyl-pentane
3. 2,2-dichloro-3-methylpentane
4. (a) 2,3-dimethyl-1-heptene, (b) 2,3-dimethylheptane

Author: CRISTOBERT C. AYATON


School/Station: LIANGA NATIONAL COMPREHENSIVE HIGH SCHOOL – LIANGA I DISTRICT
Division: SURIGAO DEL SUR
email address: cristobert.ayaton@deped.gov.ph

You might also like